Этого треда уже нет.
Это копия, сохраненная 27 марта 2021 года.

Скачать тред: только с превью, с превью и прикрепленными файлами.
Второй вариант может долго скачиваться. Файлы будут только в живых или недавно утонувших тредах. Подробнее

Если вам полезен архив М.Двача, пожертвуйте на оплату сервера.
Тред тупых вопросов №136 Roche edition 596078 В конец треда | Веб
Тред вопросов о жизни, Вселенной и всём таком.

Спрашиваем то, за что в других местах выдают путёвку в биореактор. Здесь анонимные учёные мирового уровня критически рассмотрят любые гениальные идеи и нарисованные в Paint схемы.

Предыдущий тут: >>591100 (OP)
https://2ch.hk/spc/res/591100.html (М)

Q: Можно быстрее?
A: Можно упасть в пузырь Альбукерке, NASA уже почти надула его.

Q: Я начитался охуительных историй про уфологию, че делать, нам жопа?
A: Да, тебе жопа, можешь сгонять в зогач или куда оттуда пошлют.

Q: Что будет с человеком в вакууме без скафандра / если он упадет на черную дыру / попробует ступить на поверхность газового гиганта/солнца?
A: Он умрёт.

Q: Почему бы не привязать ракету к воздушному шару или стартовать с горы?
A: Космос - это не как высоко, а как быстро, большая часть энергии ракеты уходит на разгон вбок.
Подробнее тут https://what-if.xkcd.com/58/ (английский) https://chtoes.li/orbital-speed/ (перевод)
2 596093
Паста про ебаный стыд.
3 596098
>>6078 (OP)
>>6093
Постыдный трах...
Первый - Алькубьерре.
Во-вторых, не падайте, а творите вокруг корабля изнутри (иначе не будет родственников).
В-третьих, НАСА искривляет пространство на одну десятую и использует сверхточные интерферометры, чтобы контролировать его вплоть до самого деформирующего исполнительного механизма, как рак в Антарктиде.
4 596101
>>6078 (OP)
Разве будет так резко как на видео ломатся.
5 596108
>>596073 →

>Не, не смогу, образования недостаточно, только если примерно почувствовать.


>- электроны слишком лёгкие?


>- даже если виртуальную пару растащить и сделать реальной - она сразу анигилируется об соседей?


>- впринципе виртуальную пару ЭМ полем не растащить в две реальные частицы?


>- пока будешь растаскивать и перенаправлять - результирующий импуль погасится в ноль?


>я хз короче.


Почему ЧД не колбасит в разные стороны и почему она не движется никуда от Хокинговского излучения?
image.png47 Кб, 352x102
6 596116
>>6078 (OP)
Система на третьем пике на самом деле циклическая, или там возможен только один такой перенос? Водорода в красном гиганте не так чтобы много, и его остатки ещё сильнее выгорят при эволюции этого нового голубого гиганта.

Алсо, довольно долго тупил в двойной копирайт в углу, лол.
7 596118
>>6116
Перефразируй вопрос, пожалуйста. Не очень ясно чего ты хочешь понять.
8 596119
>>596068 →
Можно. Проблема в том, что это ничем не лучше ускорения фонариком. Чтобы фармить рабочее тело из вакуума - нужно как-то запасти овердохуя энергии. А если у тебя есть овердохуя энергии - можно ускоряться обычным фотонным лучом неебической мощности.

>>596107 →

>Почему ЧД не движется никуда от Хокинговского излучения?


Потому что фотоны летят во все стороны.
Если привязать к ЧД зеркало с одной стороны (типа как двигатель Шкадова для перемещения звёзд) - она будет ускоряться.
9 596124
>>6118
На картинке нарисован перенос массы с красного гиганта слева в звезду справа, которая становится при этом голубым гигантом. Когда звезда справа выгорит и превратится в красного гиганта, пойдёт ли обратный перенос оболочки в звезду слева, которая из-за этого снова станет голубым гигантом? Или из-за изменения состава звёзд такие переносы туда-сюда станут невозможны?
10 596127
>>6124
Я вообще не представляю, что ты несёшь, тебе стоит хоть раз релевантную картинку приложить, потому, что у тебя "(C) Slow mass transfer... (c) 2014 Pearson Education Inc." и больше ничего на картинках нет. Хер знает про что ты.

Но если говорить именно о бинарных звездах, то нет, обычно утекание происходит при том моменте когда вторая звезда (по возрасту) расширяется и она дает свою материю на карлик или НЗ/ЧД. Это классика, тогда Новая происходит
image.png2,1 Мб, 844x2390
11 596128
>>6127
Я уже хуй знает, как тебе объяснить. Что тебе не понятно?
12 596129
>>6128
Нет, это тебе чего непонятно? Все же понятно нарисовано.
13 596130
Вы прослушали очередной выпуск треда тупых вопросов, плавно переходящих в тред тупых ответов. Спасибо за внимание.
14 596131
>>6108
Ну так я потому и говорю в полу бублик - в районе жопы создаём поле с градиентом перпендикулярно продольной оси корабля, а потом плавно загибаем его, что бы пары родившиеся в середине развести влево и вправо и по дуге разогнать и выплюнуть назад.

>>6119
А более массивные виртуальные частицы не рождаются, что бы больше смысла ими отталкиваться?
Например Протон/анти Протон.
Можно ещё не только их назад кидать, а ещё немножко к оси доворачивать, что бы был шанс больше что они аннигилируют прямо за кормой и дополнительно толкнут фотоном (или чем там).
15 596132
>>6116
Когда вторая звезда тоже выгорит и заполнит свою полость Роша, ничего никуда перетекать больше не будет, образуется общая оболочка, в которой обе звезды будут тормозиться и падать друг на друга.
16 596133
>>6132
Ну не всегда такой сценарий. Есть разные двойные на разных этапах эволюции. Например левая могла догореть до белого карлана, а правый синий станет красным гигантом и начнётся перекат вещества с гелиевой вспышкой.
17 596134
>>6132
Ага, ну то есть процесс не зациклен. Я просто думал, что ко времени раздутия правой звезды левая успеет схлопнуться в белый карлик.
18 596135
А ещё я буду чаще обновлять тред перед отправкой, ибо >>6133

>>6134
19 596143
Лоренцево сокращение увеличивает шансы частицы протунелировать?
20 596147
Почему небо синие(голубое)
21 596148
>>6143
Да
22 596160
>>6143
>>6148
Поясните, пожалуйста.
23 596161
>>6147
https://ru.wikipedia.org/wiki/Рэлеевское_рассеяние
По простому на пальцах - примерно так же как и призмы работают которые радуги делают.
# OP 24 596173
Нет желающих в следующий раз перекатить?
25 596178
>>6173
Ты здорово справляешься.
Я могу вернуться как это было пару лет назад, не вопрос.
26 596183

>Roche


Как это правильно читается – Роке? Роче? Рохе? Роцхе?
27 596185
Рыбников прав насчет скорости света?
28 596186
>>6183
Рош, он француз.
29 596187
>>6185
Рыбников всегда прав , но не везде.
30 596188
>>6183
Рош.
31 596189
>>6185
Судя по тому, что это какой-то ноунейм - нет. Был бы именитым учёным - можно было бы обсудить. А всякие левые бредовые теории обсуждайте у себя в вацапах или где там сидите.
32 596208
>>6078 (OP)
Анимация говно, кстати. Там шарик разваливается, а ведь он задолго до это должен растянуться. И еще раскалиться от трения.
33 596217
>>6208
ХУДОЖНИК ТАК ВИДИТ, твои аргументы инвалид.
34 596242
>>6078 (OP)
Если я оттолкнусь от МКС в сторону от Земли, меня она всё равно в итоге назад притянет?
35 596243
>>6242
Нет, у нее притяжение милипиздрическое. Ты уйдешь на другую орбиту, которая будет пересекаться с орбитой МКС в точке, где оттолкнулся.
36 596245
>>6243
И можно будет ухватиться за МКС обратно?
37 596246
>>6245
Нет, у вас частота оборотов будет чуток отличаться. Ну, то есть, когда-то может и можно будет, но ты к тому времени мумифицируешься уже.
38 596247
>>6242
МКС тебя никуда не притянет, она для этого слишком маленькая. Ты прилетишь обратно либо через виток, либо через полвитка (если оттолкнулся "вбок", например). Если оттолкнешься "вперед" по направлению движения, то хоть обратно и прилетишь, но уже немного позади станции, и будешь отставать с каждым витком. Если "назад", то впереди станции, и никогда до неё не доберешься, возмущения орбиты тебя рано или поздно сведут в атмосферу.

Точно обратно ты никогда не прилетишь, всегда промахнёшься, из-за тех самых возмущений.
39 596248
Вокруг Луны из-за масконов считай нет стабильных орбит. А что с Землей? Или там тут этим можно пренебречь, и атмосфера дает бóльшие погрешности?
40 596250
>>6243
>>6247
Я неправильно сформулировал вопрос, имел ввиду притяжение Земли а не МКС. Спасибо за ответы :3
41 596252
>>6248
Можно пренебречь, Земля гораздо более однородна, плюс её более сильная гравитация и более высокие орбиты спутников практически полностью сводят влияние масконов на нет. У нас тут наоборот, нихуёвые усилия надо приложить, чтобы неоднородности гравитации вообще замерить.
42 596254
>>6252
Эх, никакой романтики!
43 596265
>>6248
Существует общая закономерность - чем тяжелее тело, тем более оно круглое и "жидкое", во всех смыслах. Потому что самогравитация, задающая форму, всё более доминирует над любыми другими силами.

Поэтому тяжёлые планеты в любом случае должны быть более равномерны по внутренней структуре чем легкие. Звезды по той же причине почти совсем круглые по форме (если не вращаются быстро как Ахернар). А нейтронные звезды практически идеально круглые, даже если крутятся сотнями оборотов в секунду.
44 596267
>>6248

>атмосфера дает бóльшие погрешности


Среди возмущений орбиты, примерно до 1500км высоты ионосфера Земли доминирует. Дальше уже Луна и солнечный ветер влияют больше. Возмущения за счет неравномерности геоида по сравнению с эталонным эллипсоидом - совсем маленькие.
45 596268
>>6265
Чем тяжелее и компактней, т.е. ближе к радиусу Ш.
46 596307
>>6252
А как Гоги замерял разницу-то? Как он находясь на орбите может чувствовать притяжение?
47 596310
>>6307

>Как он находясь на орбите может чувствовать притяжение?


Он может чувствовать его изменение. Плотность участка планеты под аппаратом меняется - сверхвысокоточный акселерометр чувствует ускорение в ту или иную сторону.

Ну и естественно, надо уметь компенсировать остальные силы с как минимум не худшей точностью, чем у акселерометра. У гочи эту роль выполнял ёбистый GPS-приёмник, высокоточный стандарт частоты (атомные часы) и бортовой катафот, по которому презиционно измеряли орбиту с земли.

А вообще грубо наблюдать можно по отличию траектории аппарата от предсказанной, например.
48 596311
>>6248

>Вокруг Луны из-за масконов считай нет стабильных орбит


Есть же, LRO собственно по такой и крутится.
49 596313
>>6310

>сверхвысокоточный акселерометр чувствует ускорение в ту или иную сторону.


Каким образом? Он же практически в свободном падении, не отличается по траектории от аппарата в котором расположен.
50 596316
>>6313
Не в 100% свободном, на то она и микрогравитация. При пролёте над повышенной концентрацией массы она на самую писечку создаёт чуть большее притяжение, этого достаточно чтобы акселерометр триггернулся. Да, аппарат быстро придёт в свободное падение опять, но изменение всё равно почуствуется.
51 596323
>>6316
Так он реагирует на повышение гравитации. А будет ли это работать с её уменьшением?
52 596327
>>6323
Ясен красен. Просто в другую сторону.
53 596338
Останется ли от человека вообще что-то, если он как Шепард во втором Масс Эффекте упадет в атмосферу? Прост там дальше было найдено тело Шепарда, хотя он вроде как сгорел упав в атмосферу, когда первая Нормандия была уничтожена внезапно появившимися коллекционерами.
54 596339
>>6338
хуй знает что там за атмосфера была
55 596343
>>6339
https://masseffect.fandom.com/ru/wiki/Алкера

>Кора Алкеры состоит из углерода и водяного льда. Большой размер при малой плотности позволяет планете удерживать толстую атмосферу из метана и аммиака. Считается, что если бы при формировании звёздной системы Алкера обладала бы большей массой, то она стала бы ядром газового гиганта. У Алкеры есть три спутника: Улуру, Ванжинья и Байаме.

56 596473

>Согласно исследованиям, Дездемона может столкнуться с Крессидой или Джульеттой через 4—100 млн лет[7].


Речь о спутниках Урана. А как бы выглядело такое столкновение? Как я понимаю, поскольку они движутся в одном направлении, относительная скорость при столкновении будет не очень большой?
57 596482
>>6473
забей, это чисто теоретическая хуйня, реальные орбиты спутников Урана сложно предсказать, так как данных слишком мало. Вот если пошлют аппарат, которые пролетит мимо них, померит их массу, резонансы, вращения, массу колец, высчитает всю хуйню - тогда можно будет с точностью говорить.
58 596483
>>6482
Меня не что-то конкретное интересует, а просто столкновение двух спутников.
59 596484
>>6483
ну хуй знает, даже если они "медленно" будут друг в друга врезаться, тереться друг о друга, то энергия будет такая что они разогреются и развалятся. А если у них орбиты эксцентричные, то там энергии точно будет больше, если один спутник будет разгоняться из апоурания, а другой замедляться из периурания и они столкнутся так. Эти предсказания не рассказывают часто о геометрии столкновений заметь, так как мы геометрию самих спутников почти не знаем, так что там просто крутят точки или шары и потом у них симуляция говорит "1525 раз точки прошли мимо друг друга на расстоянии 1 км, 45 раз точки прошли через друг друга за время такие-то миллионы лет" а может и нет
1597178111735.png1,7 Мб, 1500x1600
60 596486
Чтобы узнать как они крутят симуляции столкновений, попробуй почитать эту статью
https://perso.imcce.fr/jacques-laskar/pdf/nature08096&s1.pdf

А если еще читать эту статью и одновременно слушать вот этот саундтрек, то воображение разыграется неимоверно.
https://www.youtube.com/watch?v=z_siqjpC6fQ
Хорошие статьи обязательно нужно читать под хорошую соответствующую музыку.
61 596489
>>6486
Не, сложна.

>они разогреются и развалятся


То есть не будет чего-то типа слияния ядер в один большой спутник, с разлетающейся шелухой из коры и всяких плохолежащих валунов с поверхности? Все разлетится?
62 596490
>>6489
да там дохера факторов - материал из чего они сделаны(один может быть из одного материла, другой из другого), внутренняя структура(есть ли там вообще ядра), геометрия тел(вдруг один спутник это вытянутая игла как оамуамуа, или как арракот состоит из двух частей, а другой спутник это крепыш-ядро), кто в кого врезается, кто тяжелее импактор или цель, вектор и скорость столкновения, имеют ли рядом стоящие спутники приливное воздействие.

Посмотри как мало мы знаем про эти спутники хотя бы на Вики
https://en.wikipedia.org/wiki/Cressida_(moon)
https://en.wikipedia.org/wiki/Juliet_(moon)
конечно в реальности мы можем знать больше чем в википедии написано, но скорее всего это маскимум
1598041016827.jpg657 Кб, 1090x768
63 596517
Так блять, я не понял, а в чем проблема сделать фотонный двигатель?
Это как ионный, только фотонный.
Берём ядерный реактор на пару сотен мегаватт, используем его энергию для высирания мощного лазера.
Все. Топливо не требуется, реактивное движение без потери массы.

Как сральное ведро, только рабочее
64 596523
>>6517

>без потери массы


С потерей массы.

>Берём ядерный реактор на пару сотен мегаватт


И никуда не летим, т.к. на каждый ньютон тяги требуется 300 ебучих мегаватт излучаемого света.
65 596524
>>6517
А ты не пробовал посчитать что получится в теории?
Если я не ошибаюсь, то тяга лазера (в ньютонах) равна мощности, деленной на скорость света. Допустим мощность реактора один мегаватт, КПД лазера 100%, а масса всей конструкции одна тонна. Сколько оно будет лететь хоть куда-нибудь?

>реактивное движение без потери массы.


Если только условно. Реально будет масса уменьшаться, иначе никак.
66 596528
Где больше рачков, на Европе или Инцеладе?
67 596529
>>6338
Зависит от скорости входа, плотности и т.п. На Земле даже угольки не долетят.
68 596530
>>6528
На дваче.
69 596531
>>6490

>кто в кого врезается, кто тяжелее импактор или цель


А почему на вот это должно быть не похуй?
70 596532
Как там дела у вазимра?
image.png3,6 Мб, 2078x1490
71 596540
У нас на сраной земляшке корону нельзя увидеть кроме как в случае полного затмения и по новостям в 2020 году из-за атмосферы, как я понимаю, а будет ли ее видно перед рассветом до прихода терминатора (не Шварца) на поверхности планеты/спутника без атмосферы? Такое-то гало поднимается, а потом яркая вспышка и само солнце уже светит?
72 596542
>>6524
Если бы я умел считать, то в этом треде бы не сидел
73 596544
>>6532
Никак. Он жрёт дохуя, и поэтому ему ёба-аппарат надо пилить. Разве что маленький вариант испытывать.
74 596545
>>6531
На дороге видел столкновения легковушки и грузовика? Имеет ли значение масса и кто в кого врезается с какой стороны и скорости?
75 596553
>>6517
Нет ни таких реакторов расчитанных на космос, ни целей ни средств.
Если тебе надо дёшево и быстро подскочить внутри СС включая облако Оорта, то пока самая годная идея сначала подлететь как можно ближе к Солнцу а потом развернуть паруса и хуярить на всех парах.
76 596555
>>6545
Кто в кого врезается - ваще никакой разницы, важна скорость столкновения, у объектов в космосе вообще нет понятия кто в кого въехал
77 596557
>>6555

>Кто в кого врезается - ваще никакой разницы


Для протокола важно.
78 596558
>>6555
есть разница на последующие траектории осколков.
79 596561
Как измеряют момент инерции космических тел? Масса понятно - по скорости вращения спутника (естественного или искусственного) по данной орбите. А момент инерции?
80 596562
>>6517
В том, что реактор не идеален и будет нагреваться. А тепло в вакууме никуда деваться не будет, при этом энергии лазер будет жрать дохуя и долго.
Лучше уж тогда использовать ярд для нагрева водорода, так хоть какая-то тяга будет.
81 596571
>>6558
Никакой

>>6557
Ну тщ сержант, там же знака нет
1598107373269.jpg59 Кб, 1024x875
82 596576
Планирую в будущем купить у Илона Маска билет на Луну, высадиться вблизи американского лунного модуля и взорвать его.

Так вот, будет ли американское правительство преследовать меня по возвращении домой?
83 596577
>>6576

>Так вот, будет ли американское правительство преследовать меня по возвращении домой?


Да.
erosimage.jpg38 Кб, 798x336
84 596578
Смотрите, что обнаружили на Эросе. Это космический сарай пришельцев?
http://near.jhuapl.edu/iod/20000503/
85 596579
>>6578
Это камень.
86 596580
87 596581
>>6579
Смотри на тень, которую он отбрасывает и на его яркость. Не похоже на камень. Я не ксенфомалити-фаг, который в каждой пупырке видит жизнь, но это очень странная штука.
88 596582
>>6577
Почему?
15981083735800.jpg55 Кб, 798x336
89 596583
>>6581
Тень ничем не отличается от тени соседних хуиток.

>>6582
Потому что ты взорвал лунный модуль.
90 596585
>>6578

>Во-вторых, он стал первым астероидом, у которого появился искусственный спутник, NEAR Shoemaker (в 2000 году), и на который спустя год этот космический аппарат совершил посадку.

91 596586
>>6585
а фотку кто сделал тогда по твоему? Астронавт карлан выскочил из аппарата и сфотографировал севший аппарат уже улетая?
92 596587
>>6586
Пришельцы же.
93 596588
>>6583
Камень квадратный и имеет рядом присоединённую секцию, которая образует мостик? Это не естественно, особенно учитвая что оно в кратере уже.
94 596589
Насколько опасен космос для человека? Вот есть радиация, а что ещё? Мне кажется космос дает четко понять, что нехуй туда лезть, но чего люди все прутся туда? Как вообще наука сейчас решает проблемы безопасности нахождения человека в космосе?
96 596592
>>6589
какой космос ты имеешь в виду? Уточни, где именно в космосе. Нет никакого абстрактного "в космосе", есть много разных конкретных мест в космосе, где условия разные и пока мы не поймем о каком месте ты хочешь услышать - не поймем как отвечать. Есть околоземная, геостационарная, окололунная орбиты, лунная поверхность, солнечная орбита на расстоянии 1 астрономической единицы от Солнца, есть марсианская орбита, орбита Венеры, орбита Меркурия, орбита Юпитера - везде разные условия и радиация.
97 596597
>>6578
Это протомолекула.
98 596598
>>6589
Россия тоже четко дает понять, что нехуй тут делать, но какие-то долбоебы тут даже живут.
99 596601
>>6598
This. Как кто-то вообще додумался жить на территории, где можно кони двинуть, тупо оказавшись на улице в неподходящее время года без средств температурной защиты?
100 596606
>>6601
Странный вопрос, однако!
101 596608
>>6601
Ты чо сука, против колонизации Марса?
Тебе дали территорию для тренировки.
102 596648
>>6517
Без задач
103 596665
Если подключить к батарее лампочку, то лампочка будет светить и нагреваться.
Если подключить к батарее резистор, то резистор будет нагреваться.
Если подключить к батарее резистор и лампочку, то лампочка будет нагреваться, а резистор нет, хотя лампочка будет светить слабее.

Как резистор умудряется уменьшать напряжение и не нагреваться?
104 596672
>>6665
Никак. Он нагревается.
105 596690
>>6672
Но нет же, не нагревается
1598177990132.jpg109 Кб, 1061x1018
106 596694
Почему космическая промышленность не вкладывает все бабки в разработку более совершенных двигателей с более высоким импульсом?

Те же клиновоздушники находятся на стадии "пререлиз".
Или вот двигатели скайлона.
Или кучи других проектов.

Они там что, не понимают, что если уменьшить стоимость вывода в несколько раз, то это будет уже новая эра в его изучении и освоении?
107 596698
Насколько стабильно реликтовое излучение?

Дистанции до источников довольно-таки приличные - подвержено ли оно ощутимому эффекту паралакса в масштабах солнечной системы? На межзвёздном масштабе?

Можно ли выбрать на карте реликтового излучения какие-то наиболее контрастные точки и юзать их для ориентации в пространстве? Практично ли держать на борту такой детектор для ориентации?
108 596718
>>6694
Кто тебе, долбоёбу, сказал, что космическая промышленность не вкладывает все бабки в разработку?
Хуею с дебилов.
109 596723
>>6690
У тебя погрешность пальца высокая. Попробуй медицинский пирометр.
110 596728
>>6694
1. Вкладывают, но химические двигатели почти достигли предела. Идёт разработка детонационных двигателей, например. Сейбр не очевидно как запилить. Других проектов нет особо, это все слабореализуемые по разным причинам фантазии.
2. Клиновоздушники это мем, они на самом деле хуевые - тяжелые, горячие и имеют смысл только в SSTO с вертикальным взлётом, а SSTO всегда хуже многоступенчатой схемы. В многоступенчатой же можно повесить два разных движка, и - о ужас - две топливных пары (тяговую внизу, высокоимпульсную вверху).
3. "Уменьшение стоимости вывода" - точно такой же мем. Аппараты стоят дороже пусков даже сейчас, и всегда будут стоить достаточно дорого, просто потому что в космосе летать сложно.
111 596749
>>6698
Интересный вопрос
cobe53ghz.gif100 Кб, 612x792
112 596753
>>6698
Очень стабильно (колебания в считанные микрокельвины), это ж изначально излучение газа, равномерно заполняющего всю Вселенную. Никакого параллакса нет, контрастных точек нет, ориентироваться по нему невозможно, да и даже пытаться это делать — быть дауном, тебе ж дали отовсюду видные пульсары, ориентируйся по ним.
1598200168793.jpg389 Кб, 1080x1255
113 596754
>>6698
Технически это возможно
114 596759
Какая программа покажет нашу Галактику с планетами, а желательно и всю Вселенную в движении?
115 596763
Можно ли теоретически сделать космо-телескоп с существующими перспективными технологиями, который не сломается и не закончится даже через 1000 лет?
116 596764
>>6763
Нет, нельзя.

Космос - место опасное, огромную чугуниевую йобу туда не выведешь, от малоточных приборов толка нет, а любая точная аппаратура сложна и ненадежна даже в условиях земляшки.

Но можно летать на шатоле и чинить.
117 596775
>>6764

>можно летать на шатоле и чинить.


Не нужно. Вместо этого нужно сразу же вместе с телескопом пускать 2 дронов роботов с руками, которые к нему пристыкованы и могут отстыковаться, осмотреть, сами поченить.
118 596776
>>6775
Лучше тогда орбитальную СТО+АЗС с дронами ремонтниками и заправщиками, телескоп то там не один болтается.
119 596777
>>6776
Смотря как далеко. Какой-нибудь телескоп в точке фокусировки солнца скорее всего будет один.
120 596779
Пока нашел
https://celestia.space/
http://spaceengine.org/
Какую посоветуете?
>>6759
121 596781
>>6779
Первую, может там поняшки есть.
122 596863
Посоветуйте какие-нибудь годные лекции/курсы/видосы на тему астрономии, вот этого космоса вашего, чтобы начать понимать, что там вообще происходит и как всё работает.
123 596872
>>6863
Хочешь понравиться одной девочке?
124 596878
>>6872
А может мальчику. Давай лучше видосы кидай.
125 596879
>>6775
А если ремонт сложный, типа замены линзы?
126 596890
>>6879
Проще новый телескоп в таком случае запустить.
index.jpg7 Кб, 225x225
127 596927
Увеличится ли подъемная сила воздушного шара, накачиваемого горячим воздухом, при снижении температуры окружающей среды?
Можно ли будет, например на титане летать на шаре наполненном воздухом комнатной температуры?
128 596937
>>6927
Да. Чем холоднее вокруг, тем выше плотность воздуха.
image.png2 Мб, 1149x893
129 596959
Почему у него сопла яркие внутри?
130 596961
)
131 596965
>>6959
Потому что так нарисовали.
132 596966
>>6965
Это же фотография
133 596971
Всем ку, добра, позитива etc
Вот я слышал, что наша вселенная может быть одной из множества других со своими законами физики и жизнью(её отсутствием)
Вопрос - Эти вселенные(очевидно включая и нашу) находятся в каком-то пространстве, как планеты, звёзды, черные дыры внутри нашей вселенной?
Ну, логически это может быть так?
134 596974
>>6753

>этот иньянь


Китаезы о чем-то догадывались?
135 596975
136 596980
>>6974
Нет. Это развёртка сферы на плоскость. В 3Д это выглядит как шар у которого половина синяя половина красная. Можешь сделать другую развёртку будет другая картина.
4r.gif178 Кб, 625x200
137 596982
>>6971

>находятся в каком-то пространстве


Да.

>как планеты, звёзды, черные дыры внутри нашей вселенной


Нет.
138 596984
>>6982

>Да.


Неверно.
139 596993
Вопрос про ВРЕМЯ
Сутки на Земле определяются 24 часами. Но почему-то именно 24, а не 100, 13 или 42.
Помнится, кто-то мне объяснял, что тут всё хитро завязано на математике и геометрии. То есть, даже если наша планета вдруг остановится, или наоборот начнёт наворачивать круги вокруг своей оси разительно быстрее, то всё равно будет 24 часовых пояса и время на 24 часа, просто планет больше не будет в них укладываться.
И вот читаю я рассказы про далёкие миры, играюв игры, смотрю фильмы-мультфильмы, и меня вё будоражит этот вопрос.
Почему 24-то?
На других планетах, получается, можно вывести формулу времени тоже основываясь на математике? Как? Если я говорю, что есть планета с сутками на 31 час или планета с сутками на 13, то это не потому что вне зависимости от размера, они вращаются одна быстрее, другая медленнее, а потому что у них математически другие параметры? Это как-то связано с тем, что метр на земле выводится тоже математически?
image.png301 Кб, 955x955
140 596994
>>6993

>24 часовых пояса


Их 37.

>Почему 24-то?


Так ПРИНЯТО. Все, больше нет причин.
Французы лет двести назад хотели метрическое десятичное время ввести 10:100:100 но не срослось, к сожалению.
Секунда была бы чуть быстрее.

На других планетах будут свои часы.
141 596995
>>6994
Слово "метрическое" забыл зачеркнуть.

быстрофикс
142 596999
>>6993
Ето не математика, ето так получилось. Шумерская, что ли, система счисления, оттуда все эти дюжины и прут (12 x 2 = 24 часа, 12 x 5 = 60 минут). Все просто спопугайничали, а потом менять было уже поперек шерсти. У леворюционных французов, вон, не получилось.
143 597001
>>6763
Нельзя, в спутнике есть и электроника, постепенно набирающая дозу и её может прошибить тяжелой частицей, и механика (например маховики ориентации), и память с ограниченным сроком жизни, и много чего ещё нестабильного, те же материалы - космос весьма агрессивная среда в целом. В общем лет 50 аппарат прожить вполне способен. 1000 - вряд ли, даже если его под это специально заточить.

Ну и 1000 лет ему просто не нужны, т.к. уже через несколько десятков всё нахуй может поменяться.

>>6879
1. У больших телескопов нет линз, у них зеркала. Зеркала легче и их проще делать при больших диаметрах.
2. Главное зеркало заменить не сильно сложнее, чем бортовой компьютер, преобразователь напряжения или маховик.
3. Но оно составляет большую часть цены аппарата, поэтому смысла мало.
144 597003
>>6999
Всегда было интересно, как так вышло, что эта система одна на всю землю. Правда не настолько интересно, чтобы это нормально погуглить. Были ли другие какие-то системы измерения времени, которые потом были вытеснены?
145 597005
>>7003
Хз, там мутная история.
https://ru.wikipedia.org/wiki/Час#История
146 597006
>>7003
Римасосы распространили вместе с футами и лигами.
147 597011
Щас видос какой-то посмотрел, нихуя не понял. Короче, вот в центре галактики находится черная дыра, да? А ещё там было что-то про квазар. Типа они пронизывают вселенную своим всплеском энергии как будто как из середины тоже. То есть, квазар и черная дыра находятся вместе посередине? Если да, то почему черная дыра её не засасывает?
148 597013
Что такое инфляция? Схуяли она появилась и закончилась? Не является темная энергия инфляцией? Не является ли инфляция костылем ебаным, чтоб теория не развалилась?
149 597015
>>7006

>Римасосы распространили вместе с футами и лигами


>распространили вместе с футами


>с футами


mein respekturen
люблю футанари
mrchangsenorchanglaughspitsmilk.gif954 Кб, 285x235
150 597016
>>7015
Содомит.
image.png2,2 Мб, 1920x1080
151 597017
>>7011
Квазар и есть черная дыра.
Когда у нее много ЖРАТ, она фонит шо твой экран мобилочки посредь ночи.
Такое было на заре формирования голактек, когда они были молодые, шутливые и жестокие.
Давно все устаканилось и современные галактики без квазаров, черные дыры посередке только чутка хавают звезды, и потому фонят слабо.
Квазары видны те, которые были давным далеко, еще с тех времен.
152 597019
Можете пояснить, почему луна улетает от земли, а титан от сатурна? Я про орбитальную механику знаю только с ksp и там если на обьект не действует никакая сторонняя сила, то он будет на своей орбите вечно.
То что тогда двигает луну от своей планеты?
153 597020
>>7019
Земные приливы ускоряют ее.
154 597021
>>7020
мда, ебать я дибил, в следущей же минуте видео поясняется причины отдаления что луны, что титана. правда, я так и не понял как именно приливные силы влияют, ну да и ладно
image.png30 Кб, 312x169
155 597023
>>7021

>правда, я так и не понял как именно приливные силы влияют, ну да и ладно


Земля вращается быстрее, таща с собой приливы, в частности с помощью суши, луну утаскивает за ними, передавая ей часть момента.
Это убивает краба замедляет вращение земли.
156 597024
>>7023
на картинке нихуя не понятно, но смог представить, как это работает. Спасибо за пояснение.
157 597025
>>7024
Соболезную.
Это самая простая и понятная схема которая идеально объясняет феномен.
158 597026
Правда, что при наличии дыхательной маски с баллоном кислорода и теплой одежды на Титане можно выжить без герметичного скафандра?
159 597027
>>7026

>−179.5 °C


Ты где такую одежду видел-то?
160 597028
>>7024
Луна поднимает прилив. Но у земной поверхности есть нехуевая вязкость, поэтому прилив "сразу" не сгладится, если убрать создавшую его силу. Земля вращается быстрее, чем Луна по орбите, поэтому приливный горб убегает из-под Луны вперед (он "не успевает" разгладиться, см. предыдущее предложение). Приливный горб притягивает к себе Луну, а поскольку он "впереди", то он придает ей ускорение, заставляя карабкаться выше.
161 597029
>>7025
Тв я и без нее понял. Просто на картинке нет суши, которая толкает воду, да и сама вода изображена фигово.
162 597030
>>7028

>он придает ей ускорение, заставляя карабкаться выше.


Он огурцачер, он понимает, что ускорение поднимает орбиту.
image.png46 Кб, 922x819
163 597031
Вот более понятная картинка с водой и землей
image.png729 Кб, 924x3141
164 597033
>>7031
НАЧАЛЬНИК ПО ОРБИТАЛЬНОЙ МЕХАНИКЕ УШЕЛ НА ОБЕД
@
ЛУНА НАМАТЫВАЕТСЯ НА ЗЕМЛЮ
@
ТИТАН ЛЕЗЕТ В УРАНУС
@
ПЛУТОН ОБЪЯВЛЯЮТ ПЛАНЕТОЙ
@
НА ОРБИТУ ЗАПУСКАЮТ ПЛАНЕТУ ИЗ МОЛЯ КРОТОВ
165 597034
>>7027
Хорошо, очень теплой одежды.
166 597035
>>7034
Ты только что скафандр.
167 597036
>>7026
>>7027
Ну если только температура беспокоит - то почему бы и нет, из какой-нибудь керамоткани одежда вполне выдержит и изолировать сможет. И на еблет нужны очёчи/маска. По сути это будет слабо отличаться от скафандра.

Главная проблема в том, что в атмосфере Титана есть всякая шняга из цианогруппы, отравишься как нехуй. И вообще много плохо пахнущих и воздействующих на человека углеводородов. Так что только герметичный скафандр и отмываться в шлюзе.
168 597037
>>7036

>очёчи


Ты что, ДУРАЧЕК, в этом слове "ё" использовать?
169 597038
>>7037
Опять ты выходишь на связь, поддельный граммарнаци? Ё там.
170 597039
>>7038
ОчОчки.

После ж, ч, ш, щ для передачи ударного гласного о пишется буква о или ё. Буква о пишется в следующих случаях.

На месте беглого гласного о в именах существительных и прилагательных, напр.: жор, обжора, прожорливый (ср. жрать), жом (ср. жму), ожог, поджог, пережог, изжога (ср. жгу, жгла); рожон (ср. род. п. рожна), шов (шва); княжон (род. п. мн. ч. от княжна), ножон (род. п. от ножны — устар. вариант слова ножны), мошон, мошонка (род. п. мн. ч. и уменьш. от мошна), кишок, кишочки (род. п. и уменьш. от кишки), квашонка (уменьш. от квашня), кошомка и кошомный (от кошма), очочки (от очки), очочко (от очко),
171 597040
>>7039
Да нет же, это разные слова! Очёчи - придуманное, но устойчивое слово, а не уменьшительное от "очки". И пишется именно так, поди найди мне "очочи" к примеру. а потом поищи "очёчи" и охуей
172 597041
>>7040
Зачем мне искать подтверждение неграмотности быдла, я это и так знаю.
Правило перечитывай до просветления, тут "о", это объективный факт, выключай манёвры и извинись. https://orfhographia.ru/orfografia.php?sid=17#pp17
173 597042
>>7041

>Зачем мне искать подтверждение неграмотности быдла, я это и так знаю.


Так "очочи" ты нигде не найдёшь, потому что нет такого слова. Вообще. Очёчи - обиходная игра со словообразованием, устойчивый эрратив если хочешь.
Ну и лингвистика - наука описательная. Говорят так, как говорят, а лингвисты едва успевают переставлять кофе в средний род.
174 597043
>>7042

>очочки


>корень -очоч-


>суффикс -к-


>окончание -и-


>очочи


>корень -очоч-


>окончание -и-


Все по тем же правилам.
Или у тебя новое слово с корнем -очёч-, либо ты насмотрелся на неграмотное быдло и зачем-то на него равняешься.
175 597044
>>7043
Я думаю у тебя проблемы с пониманием написанного.
https://ru.wikipedia.org/wiki/Эрратив
Свидетельство употребляемости - на пикчах. Или ты удаффком тоже по правилам будешь разбирать?
176 597045
>>7044

>Дебилы используют неграмотное свежевыдуманное слово.


С этим фактом я не спорил. От этого грамотным оно не становится. Пиши его правильно, вот и всё.
dv.png671 Кб, 4960x3508
177 597046
>>7033
Не шаришь в орбитальной механике. Во первых, орбита сменится как на первом пике, если дать определенную тягу точно к земле.
Во вторых, так как тяга ОЧЕНЬ маленькая, то и будущая траектория постоянно см пик 2 меняется, а поэтому этим вектором можно пренебречь. А вот вектором в сторону движения, который как раз есть, уже не пренебрежешь, поэтому луна и будет ускоряться, а следовательно и увеличивать свою орбиту.
178 597047
>>7046
почему то только второй пик загрузился, но так как он продолжения первого, то пох.
179 597048
>>7046

>отвечать на явно юмористический пост


>на картинке нарисована веревка которая наматывает луну


А ты НЕ САМЫЙ НАДЕЖНЫЙ ТВЕРДОТОПЛИВНЫЙ УСКОРИТЕЛЬ НА СТАРТОВОЙ ПЛОЩАДКЕ, да?
180 597049
>>7038
>>7040
>>7042
>>7044
Я, пожалуй, извинюсь, за доёбку. Слово глаза режет из-за неграмотности, но оно действительно в ходу.
181 597050
>>7048
Соре, не шарю в космическом самосборе.
image.png3,2 Мб, 3836x2154
182 597051
Какие бывают наблюдаемые квантовые эффекты в макромире?
183 597052
>>7051
Существование полупроводниковой электроники.
184 597053
>>7052
Лучше бы КВД упомянул, он хотя бы воочию наблюдается.
Фиг знает, полупроводники - это эффекты в микромире.
185 597054
>>7045
>От этого грамотным оно не становится.
Но это не точно.
15982030890560.png453 Кб, 600x337
186 597055
>>7003
>>6999

Моя манягипотеза, что все дело в числе 4. Если у тебя вместо цифр орехи/камешки/палочки/черточки, это будет наибольшее количество, которое глаз схватывает без деления на части, целиком, на лету. Пять палок мы уже разбиваем на 2 и 3, это не заметно, потому что очень быстро (к тому же десятичная перепрошивка). Циферблата кстати напрямую касается.
Наверно в какой-нибудь гештальпсихологии должны быть пруфы, а мой любимый филологический:

2 овцы - 3 овцы - 4 овцы - 5 овец
2 мешка - 3 мешка - 4 мешка - 5 мешков
2 солдата - 3 солдата - 4 солдата - 5 солдат
2 спейсача - 3 спейсача - 4 спейсача - 5 спейсачей

То есть в русском в итоге вывелось различие между "несколько" и "дохуя". А у шумеров в итоге вывелось число, делящееся на 4 и достаточное для хозяйственных нужд. инб4 пять пальцев - дохуя у тебя в жизни ситуаций, когда надо сосчитать пальцы? вот и у шумеров не дохуя.
187 597060
Почему черные дыры не зарастают?
188 597061
>>7060
Они "зарастают" в теории благодаря излучению Хокинга, но испарение занимает бесчисленное количество времени.
Это область пространства-времени обусловленная колоссальной гравитацией, эта масса находится на дне гравитационного колодца и ей не выбраться с него, т.к. даже свет не может покинуть эту область.
189 597072
Почему международный стандарт массы КИЛОграмм? Почему все единицы СИ это %единицанейм% а масса это кило+%единицанейм%? Почему не взяли Мась условную за стандарт без метрической приставки и уже от нее плясали? Был бы у нас Мась как килограмм, миллиМась как грамм и килоМась как тонна.
Как-то логика не укладывается.
Понятно, почему килограмм взяли, литр воды измерить легко, но почему он кило- то?
190 597088
>>7072
Потому что грамм звучало как граф во французком. А это по буржуйски и негоже в равноправной, пост-революционной франции иметь меру более высокую по статусу, чем другие. Вот и добавили приставку кило. А потом прижилось.
191 597092
>>6959
Отражение вспышки от сеточки аэратора в движке.
https://physics.stackexchange.com/questions/25962/what-is-lighting-up-space-shuttle-endeavours-main-engines
после 19 перезвонить.png30 Кб, 970x1124
192 597093
Может ли быть крупная двойная луна у планеты? Например, у планеты уровня Уран и больше (будь то Сатурн или Юпитер), спутник уровня Луны (земной) или вообще Титана, но спутник двойной, вращающийся парой, центр масс пары которого крутится по орбите родительской планеты?
193 597094
>>7093
Очень далеко от планеты. Не в нашей солнечной системе.
194 597098
>>7093
Такое разрушится приливными силами. Тот же эффект, благодаря которому почти все крупные спутники находятся в приливном захвате (когда оборот вокруг своей оси равен по продолжительности обороту вокруг центрального тела) и обращены к планете одной стороной. Чем дальше от центрального тела, и чем меньше расстояние в паре, тем эффект слабее, но все же сомневаюсь, что такая парочка переживет 4 миллиарда лет в масштабах планета - спутники (вот Солнце - планеты может).
195 597109
>>7093
Схема взаимодействия Януса и Эпиметея.жпг
gaaaaaayyyy.png89 Кб, 500x500
196 597113
197 597118
>>7109
Во-первых, эти пельмешки не сравнить с размером Луны или Титана с Ганимедом. Во-вторых, ЭТО космическое ДРУГОЕ - у них ведь не общий центр масс, а нахлёстная рокировочная орбита
>>7098

>Тот же эффект, благодаря которому почти все крупные спутники находятся в приливном захвате (когда оборот вокруг своей оси равен по продолжительности обороту вокруг центрального тела) и обращены к планете одной стороной.


Мне почему-то кажется, что вращательный момент в такой паре будет более сильным и влиятельным, чем стопорящий эффект, вызывающий приливный захват
198 597128
Напомните пож какой угол плоскости вращения Земли вокруг Солнца к плоскости диска Млечного Пути, если есть картинку буду благодарен.
200 597134
>>7130
Здорово, то есть в летнее солнцестояние Земля между Солнцем и центром Галактики на прямой линии, если правильно понял.
201 597136
>>7134
На самом деле наоборот. См. 4 пикчу. После осеннего равноденствия мы оказываемся в Стрельце, примерно во время зимнего солнцестояния.
Первая пикча не такая информативная, четвертая лучшая в наборе, на нее ориентируйся.
202 597137
>>7136
DISREGARD THAT I SUCK COCKS

>>7134
Да-да, ты все правильно понял.
203 597176
Сколько всего галактик в видимой и невидимой части ?
204 597215
>>7176
Два триллиона.
205 597236
Помнится, кто-то писал что на поверхности Сатурна g таково, что его не отличить от земного. Это навело меня на мысль о создании сеттинга на поверхности небольшого газового гиганта (не обязательно самого Сатурна, можно планету и поменьше), где сама планета по факту газовый гигант, но у неё на поверхности почему-то есть поверхность. Твёрдая, каменистая, с растительностью и водой, то есть обитаемая, но там корочка коры планеты небольшая, пара километров, и каким-то научно обоснованным образом, чем ниже ты бы продвигался по толщине этой коры, тем более рыхлой становилась бы толща, а потом, в самом низу коры, где кора резко обрывается и начинается условная тёплая пустота, ведущая вниз, к ядру, и конвекционными потоками оттуда поднимается лёгкая пыль и наливает на кору. Местами иногда куски налипшего падают обратно вниз, в ядре "перевариваются", а затем пылью обратно липнут к поверхности со внутренней стороны. Ну и вопрос - какого размера тогда должна быть планета, чтобы получить при критерии гравитация на поверхности как на Земле плюс-минус?
Луны у неё могут быть? Повлияли бы?
206 597268
>>7019

> Можете пояснить, почему луна улетает от земли, а титан от сатурна?


Земля - это не конкретная точка с гравитацией. Это охуенных размеров сборище точек, сиречь элементарных частиц, у каждой свое притяжение, и каждая отдельно взятая часть планеты тоже обладает своей гравитацией. Просто если в небольших по времени масштабах это можно считать единой системой, то в более крупных Земля вертится и передает энергию вращения с более близкой к Луне стороны на саму Луну, перемещая ее на более высокоэнергетическую орбиту - то есть, повыше. Со временем Луна замедлит Землю настолько, что будет висеть на новом земном геостате и ускоряться перестанет. Но к тому моменту она с куда большей вероятностью улетит в ебеня либо погибнет при расширении Солнца.
изображение.png13 Кб, 791x281
207 597282
В небе вижу пикрил. Что обведено красным и синим? Вроде как синее это юпитер, не помню точно, но вот красное?
image.png970 Кб, 1560x780
208 597284
>>7236

>поверхности Сатурна


Это где она такая?

> Это навело меня на мысль о создании сеттинга на поверхности небольшого газового гиганта


Ты только что Беспин.
209 597285
>>7236

>Ну и вопрос - какого размера тогда должна быть планета, чтобы получить при критерии гравитация на поверхности как на Земле плюс-минус?


Гравитация планеты не зависит от ее размеров. Существуют такие планеты, которые размером с землю, но при этом имеют чудовищную гравитацию, что там прямоходящие никогда не появятся.
210 597289
>>7285

>Гравитация планеты не зависит от ее размеров


Эм, чтоа? Обратно пропорциональна квадрату радиуса, прямо пропорциональна массе же (для достаточно сферически-симметричных планет, на практике радиуса в ~250 км уже достаточно для этого).
211 597290
Посчитайте, пожалста, какого радиуса будет планета из чистого осмия чтоб на поверхности был 1Г?
212 597291
>>7290
Лучше из кротов.
213 597292
>>7291
Из кротов будет планета-бульон.
214 597294
>>7290
Объем шара равен 4/3 π r^3. Масса: М = 4/3 π r^3 ρ. Гравитация на поверхности будет G M / r^2 = 4/3 π G r ρ. Подставляешь значения констант, решаешь относительно r.
215 597295
>>7290
~158 км
216 597296
>>7295
У меня получилось r ~ 1550 км. Кто из нас ошибся на порядок?
217 597297
>>7296
Я. Ноль проебал.
1552 км
218 597298
Ради интереса, для кротов у меня получился радиус около 35 000 км (предположения: плотность крота примерно равна плотности воды, кроты несжимаемы).
219 597299
>>6561
Бамп вопросу. Вот откуда он возник:

>Предположительно Ганимед состоит из трёх слоёв: расплавленного ядра из железа или сульфида железа, силикатной мантии и внешнего слоя льда[5][51] толщиной 900—950 километров. В пользу этой модели свидетельствует малый момент инерции, измеренный во время облёта Ганимеда «Галилео» — (0,3105 ± 0,0028)×mr2[5][51] (момент инерции однородного шара равен 0,4×mr2, а меньшее значение коэффициента в этой формуле говорит о том, что плотность растёт с глубиной).

220 597302
>>7296
>>7297
Немного меньше луны, а гравитация как на Земле, охуеть.
А будет у этого осмиевого шара тектоника? Точнее так - она может быть?
221 597317
>>7282
Бамп.
222 597320
>>7295
>>7296
>>7297
А сколько брать плотности у Сатурна, чтобы 1g получилось у него на поверхности?
223 597322
>>7302
Тектоника возникает не от состава, а от какого-то источника энергии, когда изнутри прет тепло, растапливающее все нахуй, а снаружи планета застывает, потому что получает от Солнца недостаточно энергии. Так что надо либо брать слаборадиоактивный изотоп осмия (кандидат тут только осмий-186), либо вешать шарик спутником на орбиту с заметным эксцентриситетом и/или наклонением к экватору центрального тела.
224 597348
>>7289
Масса же зависит не только от объема, но и от плотности.
225 597352
>>7322
а как должна переть энергия, чтобы внутреннее тепло грело планету без внешнего источника энергии? или такого слабого, чтобы не только не грело, но светило слегка. как солнце для плутона, например
226 597353
>>7320
~602 кг/м³
227 597390
>>7352
Хорошо должна. Но разогрев из-за ядерного распада вполне себе значимый источник энергии. Не уверен, что осмия-186 хватит, все же период полураспада ну ОСНЕ большой. С другой стороны, если прям всю планету из него слепить, то может его будет настолько дохуя, что и хватит. Считать надо.
228 597393
>>7317
https://yandex.ru/search/?text=карта звёздного неба онлайн
229 597394
>>7348
Но ведь зависит. Чтобы гравитация оставалась постоянной "при увеличении радиуса", нужно чтобы плотность (причем всей планеты в среднем, а не "свежедобавленного вещества) падала обратно пропорционально радиусу.
230 597400
>>7390

>Считать надо.


V≈1.56×1010km3
m = 352,716,000,000,000,000,000kg = 3.53x1017kg = 3.53x1020g
mm = 190,23 g/mol
1.12e+42 атомов осмия
Полураспад альфа - 5МэВ.
Период полураспада - 2х1015 лет.
5е+41 атомов осмия за 2х1015 лет выделят по 5 МэВ энергии.
25е+41 МэВ = 16e+22 джоуля
Примерно 10 мегаджоулей энергии в год.

Площадь поверхности 30 миллионов квадратных километров. Реликтовое излучение приносит 1мкВт/м3
Реликтовое излучение сообщит 30 мегаджоулей в год.

Выбери другой изотоп/атом.
231 597401
>>7400
С другим хуево, там по условию ОП осмий хотел. Все остальные слишком уж радиоактивные. Значит надо приливными силами как-то.
232 597402
>>7401
Ну тащемта мы про тектонику говорили, так что сугрев нужен, можно что-то и другое кроме осмия, тяжелых атомов еще немало.
Инб4 планета из оганессона распидорашивающаяся что твоя сверхновая в первую микросекунду.
233 597403
>>7402

>можно что-то и другое кроме осмия


Вообще, конечно, можно, но по условию нельзя:

>какого радиуса будет планета из чистого осмия

234 597404
>>7403
Я разрешаю не из осмия, я просто выбрал что поплотнее.
>>7290-анон
Мы уже посчитали что надо, теперь другой вопрос возник же - чтоб ОСНЕ плотная планета, но еще и с тектоникой. Хтонично будет выглядеть.
Бонусом было бы низкое альбедо, но плотные пепяки обычно металлы и будут сиять.
235 597405
>>7404
Ну тогда сыпани классического калия-40. У него период полураспада в миллион раз меньше, но все равно это больше миллиарда лет.
236 597407
>>7405
Меня смущат что я взял энергию альфа-распада в 5МэВ, она всегда такая что ли? Это и будет тепловая энергия разогрева недр?

Тока это, КАЛий легкий дохуя же, задумывалась изначально мелкая зловещая планета из плотного металла чтоб ухбля, пот на лбу выступал.

Хтонические планеты такими могут быть, или там камня налипает дохуя? Кто-то кукарекал про алмазные дожди на горячих юпитерах, может после сдува атмосферы остаться поля алмазов?
237 597408
>>7407

>Меня смущат что я взял энергию альфа-распада в 5МэВ, она всегда такая что ли?


В душе не ебу, я твоим рассчетам поверил наслово.

>Это и будет тепловая энергия разогрева недр?


По идее да, вряд ли что-то из продуктов деления сможет пробиться наружу через сотни километров осмия и просто так улететь. Все должно в тепло перейти.

>Тока это, КАЛий легкий дохуя же, задумывалась изначально мелкая зловещая планета из плотного металла чтоб ухбля, пот на лбу выступал.


Тогда уран-238.

>Хтонические планеты такими могут быть, или там камня налипает дохуя?


Да никто не знает, есть только более-менее обоснованные теории. Но что естественная планета из чистого осмия (даже если с небольшими примесями урана или калия) невозможна, тут я готов анус поставить.
image.png265 Кб, 480x360
238 597409
>>7408

>тут я готов анус поставить


Это сайфай уже больше и эксперименты в уме "а что если". Из кротов планет тоже нет (вроде бы), а обсудить интересно.
239 597410
>>7408
Мы может не с той стороны подошли. Я выбрал самый тяжелый элемент, а ведь может быть какие-нибудь молекулы/сплавы могут быть плотнее, нет?
инб4 нейтрониум
240 597411
>>7409
Ну если так, мои рассуждения такие, что в любой металлической планете будут большие (грубо говоря, половина) примеси всяких кислородов, сер и прочих неметаллов. Химически все это отлично связывается в оксиды и всякие соли, а разделение требует нечеловеческих усилий. Какому-нибудь солнечному ветру такое не под силу, он только атмосферу может сдуть, ну так она и без того с металлами не связана.
241 597412
>>7411
А что если был инцидент сопоставимый с тем, что гипотетически произошел с Психеей? Была ОСНЕ большая планета, все тяжелое говно утонуло, оксиды-хуиды вытеснились чистыми расплавами, а потом кто-то по ней уебал, или рядом нейтронки слились окропив атмосферу и камень свежей струей урины тяжелых металлов и сдув все что легче железа?
242 597414
>>7410
Из вики:

>Самые плотные вещества при стандартных условиях — металлы платиновой группы (рутений, родий, палладий, осмий, иридий, платина). Имеют плотность 21400—22700 кг/м³.


Скорее всего имеются в виду вещества вообще, а не только простые. Ну и чисто логически, плотность определяется массой атомов (масса растет куда быстрее объема, поэтому плотность нарастает) и межатомными расстояниями. Для кристаллов, например, типом решетки. Даже если там бывает что-то плотнее чистого осмия, то это наверняка какой-то сплав все тех же металлов. Явно уж не оксиды какие-нибудь, прости господи.
243 597415
>>7414
А мы один момент для сугрева забыли.
Планета не будет представлять из себя однообразный шар с одинаковой плотностью и давлением. В центре скукожит атомы поплотнее.
Заспавненный равномерный шар из осмия осядет на километров 10 сразу разогрев при этом внутренности.
244 597417
>>7412
Ну хз. Но чтобы было такое ядро из осмия, нужно чтобы были соответствующие слои из более легких металлов, которых куда как больше (того же железа должно быть на порядки больше). Что-то исходная планета по массе уже в звезду превращается, такими темпами.
245 597420
>>7412
Железо всего в 3.5 раза тяжелее камня. Не думаю, что, то что сдуло камень не сдует железо, и соответственно всю планету целиком. Да и нет особо в космосе катаклизмов способных сдуть камень. А если бы и могли это 1 на 1000 что сдует только камень.
Вот трансмутировать нейтронкой каменную поверхность в металлическую может быть. Но как ты понимаешь это скорее будет какое-то говно из разно-разложившихся элементов, где вся таблица элементов. В итоге это скорее всего превратится в камни только не в силикатные, а какой-нибудь полоний-мышьяк.
246 597422
>>7420

>трансмутировать нейтронкой каменную поверхность в металлическую


Я чот сильно сомневаюсь. Если идет такой поток нейтронов, то планету просто испарит нахуй. Ну, то есть, вещество может и трансмутируется, но планету оно уже составлять не будет, как мне кажется.
247 597423
>>7394
Тут в соседних постах планету из чистого осмия обсуждают, а ты что-то про плотность несешь.
248 597440
Опа, тут как раз про кольца Сатура в ОП-посте.
Какой объект нужно разрушить на орбите Земли, чтобы появились кольца, достаточно видимые с поверхности нашей планеты? А можем ли мы, например, прямо завтра взять, и грамотно направить ядерными взрывами спутники Марса в сторону Земли, чтобы они разрушились и образовали видимые кольца?
249 597449
>>7440
Тротила не хватит. Едва ли как-то все наши ядрёны изменят орбиту спутников марса, и уж тем более переместит их к земле.
А если мы говорим о мимо-метеоритов, то их слишком мало, и мы вряд ли их сможем затормозит на орбите.
Кольца в принципе возможно создать, но не при текущем развитии, да и ближайшие сотни лет вряд ли. Нужно кучу материала отбуксовать, а это дохрена энергии. Где эту энергию брать я даже не представляю. Ядерные буксиры должны быть массовым продуктом и скорее всего даже если им хватит тяги, ядерного материала может и не хватить. А в итоге ненужная хрень, лучше бы звёзды покоряли.
250 597452
>>7440

>А можем ли мы, например, прямо завтра взять, и грамотно направить ядерными взрывами спутники Марса в сторону Земли, чтобы они разрушились и образовали видимые кольца?


Направить-то можем, только не взлетитулетит. Да и вдвоём Деймос с Фобосом не смогу создать колец, чтобы было К Р А С N В 0. Нужно распидорасить что-то не меньше Плутона.
Тем более кольца будут осыпаться на поверхность, но это уже совсем другая история
251 597457
>>7452

>что-то не меньше Плутона


Ты ебанулся, даже у Сатурна кольца весят в триста раз меньше.
252 597465
>>7440

>направить ядерными взрывами %хуйнюнейм% к %планетанейм%


Лолнет. Просто для понимания, один лишь Фобос весит с десяток миллионов тонн, а Деймос раз в сто больше.
253 597468
>>7465

>Фобос весит с десяток миллионов тонн


Десять триллионов.
254 597472
Почему бы не ударить по Луне ядерными ракетами? Глядишь и появится там своя атмосфера.
sage 255 597474
>>7465

>а Деймос раз в сто больше.


Чо несёт? У тебя Деймос тяжелее Фобоса? Фобос в 7 раз тяжелее Деймоса
256 597486
>>7472
Из чего?
257 597510
>>7474
Он другую солнечную систему нароллил, видимо.
258 597556
>>7468
Кстати, здание МГУ весит около полумиллиона тонн, для сравнения.
259 597562
>>7452

>можем ли мы, например, прямо завтра взять


>Направить-то можем


Из какого века капчуешь? Луну заселили? На Марсе город построили?
ayy lmao in the distance.jpg25 Кб, 500x375
260 597564
>>7562

>12020 г.э.


>до сих пор даже не начали колонизировать свою солнечную систему

261 597601
>>7562
Всё построили, всё наладили! Правильную тогда Рогозин программу выбрал.
262 597656
Допустим совершаю 10-50 запусков на орбиту, чтобы собрать там корабль и ещё 1000 запусков чтобы доставить топливо и окислитель.
Что мне помешает разогнаться на построенном корабле например до 90% скорости света?
263 597657
>>7656
удельный импульс
264 597658
>>7657
Что удельный импульс?
Выходящие из сопла газы будут толкать ракету пока не кончится топливо и окислитель.
265 597659
>>7657
Как он помешает?
266 597660
>>7656
Физика.
15887887274540.jpg79 Кб, 741x561
267 597672
>>7656
Ну допустим у тебя даже не хим. топливо, а ксенон и удельный импульс 15000. Ну или водород и газофазный ЯРД. Пустой корабль весит 1000 тонн, ксенон в нём 100 тыс т., у нас ультра-баки будущего. Выкипанием, выгоранием, охладом и прочим ненужным задротским говном пренебрежём. А разгоняешься ты посреди абсолютного нихуя, так что потерь на не-мгновенность у тебя нет. При таких условиях ты разгонишься аж до целых невероятных сумасшедших 700км/с. Можешь разбить на ступени, ситуация принципиально не изменится.

>>7658
Проблема в том, что уравнение Циолковского имеет экспоненциальный характер. Так что все претензии можешь слать этому злобному старикашке.
268 597677
>>7672
Инбифо "что мне помешат долить еще сто миллиардов тонн топлива".
269 597678
>>7656
Забиваешь в поисковую строку гугла
V * ln K
Где v-скорость истечения топлива
K-во сколько раз масса корабля с топливо больше массы пустого корабля
и получаешь скорость корабля по истечению топлива
при скорости истечения 5км/с (водород-кислород из супир-трупер двигателя)
и K=1000 000 (ну очень много раз запускали ракету-дозаправщик)

получим 0.00023 скорости света
270 597680
>>7672
а если взять хуйню на пылевой плазме или еще хуже?
271 597684
>>7677

>что мне помешат долить


Гравитация, лол.
Скажем при УИ 23,5 км/c (2400 секунд) и "волшебном" преобразовании топлива, т.е. если у нас есть только ПН и топливо, а баки и двигатели весят ровно 0, и разгоняем мы ажно целый килограмм ПН,то где-то к 1400 км/c (менее 0,5% с) масса топлива превысит массу Нептуна. У которого скорость убегания как раз те самые 23,5 км/c.
Можно конечно поигратся с цифрами, но фнудаментально это ничего не изменит - пока скорости истечения ограничены сотнями-тысячами км/c, скорости сравнимые со световой, скажем 10% и более, будут требовать таких запасов топилва, что они будут удовлетворять МАС-овскому определению планеты со всеми вытекающими.
272 597685
Обладая современными технологиями обнаружения экзопланет, на каком максимальном расстоянии мы смогли бы обнаружить Землю, наблюдая со стороны?
273 597687
>>7685
Вопрос слишком расплывчатый, поэтому ответов несколько.
Гравитационным микролинзированием - как минимум с десятков тысяч светолет, но только если повезёт.
Транзитным методом - с тысяч, зато он универсален.
Обнаружить объект размером примерно с Землю и нагадать на кофейной гуще что он в обитаемой зоне данной звезды - сравнительно легко. Понять что там есть кислородная атмосфера, вода, и т.п. - намного сложнее.
Надо понимать, что современные технологии обнаружения развиваются оче быстро, и что первую экзопланету транзитным методом нашли всего 18 лет назад, а массовому поиску едва с десяток лет, по сути.
274 597688
>>7687
Разве найдены планеты на таком же расстоянии от звёзд как земля? В основном же горячие Юпитеры, на расстоянии меньше орбиты Меркурия.
276 597690
>>7689
Это гравитационное микролинзирование, без малого 25 тыс световых лет, 4 массы Земли.
277 597691
>>7688
В смысле вообще, а не на больших дистанциях от нас? Полным-полно, есть даже совсем близкие кандидаты.
278 597692
>>7689
Совсем другие параметры масс и всего прочего. Да и расстояние значительно меньше 1 АЕ.
279 597694
>>7692
Ну какие другие? Это всего-то 4 массы. Вполне себе не гигант, обычная суперземля, которых куча. Это был пример далёкого обнаружения суперземли, а не похожести на Землю.

>расстояние значительно меньше 1 АЕ


Это имеет значение лишь для транзитного метода. Разные методы имеют разную чувствительность для разных расстояний. Каким-то желательно поближе к звезде, каким-то (например прямое наблюдение) наоборот подальше.
280 597697
>>7687

>зато он универсален


Там тоже надо, чтобы плоскость орбиты экзопланеты была почти параллельна нашему лучу зрения.
image.png8 Кб, 771x375
281 597701
Итак здравствуйте аноны, не знаю куда конкретно написать по данному вопросу поэтому насру здесь.

Время было около 8ч по мск, 48 град северной широты, центральная россия, 29.08.2020 , в небе пролетело 2 пары объектов с интервалом около 3 минут абсолютно беззвучно, дело происходило в городской черте и засветка неба была достаточно велика и при этом была возможность их разглядеть, примерно 3/4 небосвода объекты преодолели секунд за 5-6, нет это были не спутники, так как диаметр свечения был слишком велик для спутников, светимость порядка небольшого метеора в верхних слоях атмосферы, цвет свечения что то типа коричнево-красного, самое интересное это траектория первой пары объектов, пикрил(красными линиями траектория черные точки приблизительный размер объектов как я их видел),

вторую пару я проебланил и увидел только когда она уже подходила к горизонту, и траекторию оных нарисовать не смогу,

При том что один объект первой пары имел синусоидальную проекцию траектории на мои глазищща, он летел паралельно второму,т.е. не отставал

ИТАК ВОПРОС
ЧТО Я МОГ ЛИЦЕЗРЕТЬ?
282 597708
>>7701
Не стоит вскрывать...
this is getting out of hand.jpg62 Кб, 1280x720
283 597712
>>7701
Ты видел НЛО! Неопознанные летающий объекты!
Скорее всего этими объектами были китайские фонарики летящие на относительно низкой высоте при большом ветре.
284 597717
>>7712
Фонарики намного ярче при такой высоте и имеют четкие формы,вечер был безветренным, буквально, а это просто размытое облачко темно красного света
285 597718
>>7717

>вечер был безветренным


То показывается погода на поверхности, на 200м не меряют ветра, где они летают.
286 597719
>>7718
в любом случае, с той стороны откуда они летели была волга, за волгой безжизненные выжженные пустоши, я видел фанарики китайские и низко и высоко и могу их отличить от неебической летающей хуйни, поэтому с уверенностью отвергаю предположение на правах наблюдателя данной неебической летающей хуйни
image.png3,3 Мб, 2000x1333
287 597720
>>7719
в интернете отыскал релевантную фотографию, но там видать выдержка большая, и объект длинноватый и яркий на зуме однако цветность передает отлично
288 597721
сука вот блять заказал бы я ебаную дифракционную решетку хоть блять бы спектр был чо там горело если горело вообще
289 597722
>>7720
На этой фотографии самолет.
290 597723
>>7722
перечитай первый мой пост
погляди на траекторию
прочитай про бесшумность
+ проблесковые маячки на гражданский и частных рейсах мигают с переодичностью
+самолет гражданский не развивает такую скорость и не имеет такой размах крыльев в угловом размере на достаточной высоте чтобы не слышать его
291 597724
первая мысль в голове была когда я это увидел, тоже была про самолет, однако ни подтвердить , ни опровергнуть , у меня не получилось , потому что думалка не сработала моментально и я не сообразил обратить внимание пропадает ли свет звезд в пространстве между источниками света (закрывает ли брюхом самолет/нëх их)
292 597726
>>7723
Значит ты никогда в жизни самолеты не видел.
Это самолет.
Дело закрыто.
293 597728
ну и чтобы внести какую-то ясность в расстояние между источниками света первой пары могу сказать что в наиболее дальней друг от друга позиции растояние между ними было равно приблизительно в раза полтора-два больше диска луны
294 597729
>>7726
щас пойду и чекну рейсы в тот промежуток времени
295 597730
с телефона фр24 неудобно пользоваться, посему попрошу тебя просто собрать свои доводы воедино
296 597732
скачал фр посмотрел за вчера с трехчасовым запасом вперед-назад
ничего зафиксированного фдайтрадаром над местом наблюдение не летало
Стикер191 Кб, 500x500
297 597734
>>7729
Малая и военная авиация на флайтрадаре не отображается.
298 597759
Я один никогда не воспринимал солнце как звезду?
299 597760
>>7759
Когда ты задаешь вопрос "я один..." - ответ в 99.999999% случаев "нет".
Подавляющее большинство людей не воспринимало Солнце за звезду, да и сейчас полно таких.
301 597762
>>7760
Я просто всегда читал про нейтронные звезды, чёрные дыры, а потом осознал что мы крутимся вокруг ёбаной звезды, изучай не хочу, хочу изучать говно в 99999млн км

Как же хочется солнечный телескоп
302 597763
>>7762
Кто "не хочу"? Солнце - самая изученная звезда во вселенной. Солнечных обсерваторий жопой жуй, постоянное наблюдение за всеми видами активности.
303 597771
>>7761
скорость малая у спутников на видео, те же преодолели 3/4 небосвода за секунд 6
304 597772
>>7771
Так это не спутники.
305 597773
>>7771
да и траектория очень странная у одного из объектов
306 597774
>>7771
Отражение фонарика может развивать сколь угодно высокую скорость.
307 597779
Кароч придумал новый вид корабля. Выводим на орбиту обычной ракетой. Дальше ускоряем ионными двигателями. Рабочее тело берём прямо из космоса. Энергию от солнечных батарей или от ядерного реактора. Таким образом постепенно поднимаем орбиту. Можно ускоряться очень долго вплоть до 20% скорости света.
308 597783
>>7774
отражение обо что?
309 597788
Поясните за пардокс атмосферы.
Есть одна Земля - о её атмосфере, жидкой воде и биомах с форой и фаунов, достоверно известно.
Есть Титан - у него есть атмосфера более плотная, чем земная, хотя Титян и меньше, и легче
Есть Свинера - у неё масса и размеры боле менее как Земля, но атмосфера плотнее и толще.
Есть Марс - с рябой и кривой, но атмсоферой имени блохастой собаки.
Мы знаем, что жидкость на поверхности регулируется атмосферой, и что при определённом наборе масс-объёма, жидость не будет выпариваться в космос, а останется на планете, ибо пойдёт в облака и в осадок, вернувшись к истокам. Атмосефра же защищает от межзвёздной и местносолнечной радиации и особенно ветра, а следовательно и от радиолиза. Но всё это только при наличии достаточно толстой атмосферы.
Но почему так?
Получается, налей мы на Титан мало атмосферы, он был был лоховским Ганимедом. Налей мы на Марс дохуя атмосферы, там и вода была бы жидкая, и яблони бы цвели. Да даже на земной Луне если вообще дохуя атмосферы налить, можно получить атмофсеру, который и лунной гравитации как раз,и солнечный ветер не страшен и земная гравитация.
Получается, существования атмосферы поддерживается... существованием атмосферы?
310 597792
>>7779

>новый вид


Роберт Бассард из-за тебя снижает крутящий момент Земли.
311 597793
>>7760

>Когда ты задаешь вопрос "я один..." - ответ в 99.999999% случаев "нет".


Вот, сука, двачую. Меня одного такое бесит?
Стикер191 Кб, 225x225
312 597804
>>7793
Содомит!
313 597812
>>7783
Об лёгкую облачную дымку. О провода. О много что.
Алсо насколько я помню у вас там на волге какой-то видеоблохер своим фонариком всех слепит и облака и волгу и т.д.
314 597841
>>7788
Тут важен баланс массы планеты, состава атмосферы, и ее температуры. Чем толще атмосфера, тем (при некоторых условиях) выше температура планеты и самой атмосферы. Чем выше температура- тем ближе скорость движения молекул газа ко второй космической, и соответственно тем быстрее атмосфера съебывается.
315 597842
>>7841
Еще ОСНЕ важна геологическая активность. Если вулканы постоянно подпердывают дополнительным газком, то атмосфера даже при довольно быстром рассеивании сможет оставаться плотной.
316 597845
>>6994
кроме марса
317 597853
>>7812
эт дурачки с прожекторами вроде цирк или еще что то, на облаках часто бывает да, но тогда и облаков не было, звезды прекрсно были видны
318 597857
>>7788
Атмосфера Земли биогенная, если б не жизнь она была бы совсем другой. И тащемта была - в ранние моменты её существования, да и после кислородной катастрофы и давление и состав долго были непохожи на то что сейчас.
319 597858
>>7788

>Получается, существования атмосферы поддерживается... существованием атмосферы?


Ну да. Что тебя удивляет? Титан перетянул на себя дохуя при формировании, поэтому он такой. Венера ещё больше, поэтому она такая. А уж газовые гиганты перетянули охуеть сколько. Выдувай - не хочу, солнце раньше лопнет.
320 597860
>>7299
Бимп. Ну что же вы, анонимные ученые?
321 597865
>>7860
Взял бы да погуглил.

Забиваешь в гугл Ganymede moment of inertia, шаг 1: https://en.wikipedia.org/wiki/Moment_of_inertia_factor, шаг 2: https://en.wikipedia.org/wiki/Darwin–Radau_equation

Момент инерции для тел, находящихся в гидростатическом равновесии, т.е. приобретшим форму эллипсоида вращения под воздействием собственной гравитации и центробежных сил, можно вывести из соотношения между массой, скоростью вращения и полярным сжатием.

Утрированный пример — если бы 99,9% массы Ганимеда было сосредоточена в точке в его центре, а остальная планета состояла из сахарной ваты, то никакого полярного сжатия бы вообще не было — гравитационное поле было бы практически идеально сферическим. И наоборот, если бы он был полностью однородным, то полярное сжатие бы полностью соответствовало теоретическому для однородной несжимаемой жидкости.

Зная, где именно Ганимед находится между двумя этими крайностями, можно примерно так почувствовать, что у него внутри.
322 597874
>>7865
Спасибо.
323 597913
Сколько сантиметров слоя атмосферы с юпитера надо снять, чтобы покрыть все планеты, спутники, карликовые планеты солнечной системы атмосферой с плотностью одной земной?
324 597921
>>7913
Большая часть всего этого зоопарка атмосферу с плотностью земной неспособна удержать в принципе, хоть ты весь юпитер туда надуй.
325 597945
>>7845
Что значит "кроме Марса"?
На марсе тоже будут свои часы.
Предполагается что 37 минут будут идти как двадцать пятый час.
После 23:59:59 пойдет 24:00:00 и так до 24:37:00, после чего 0:00:00
326 597970
Почему СССР построил космодром в Казахстане, а не в более южной Туркмении?
327 597972
>>7970
Надо тысячу с лишним километров своей территории для зон падения ступеней. В китай бы падало, что не айс.
Мне так кажется.
328 597982
Придумал как добывать газ. Берём очень большой пылесос и засасываем газовые гиганты.
Ну как вам идея?
329 597983
>>7982
Хороший план
330 597984
>>7982
Годно. Приступай.
image.png74 Кб, 304x166
331 597986
Возможно ли построить город на колцах сатурна? Там ведь много астеройдов по размерам как раз подходящих для дома, они близко и их можно соединить дорожками. Представьте как там будет красиво эх... вместо солнца будет сатурн. Готовы купить там домик?
332 597987
>>7986

>Возможно ли построить город на колцах сатурна?


Нет.
333 597989
>>7987

>>пук пук

Стикер191 Кб, 500x500
334 597991
>>7989
Что и ожидалось от ГАЗОВОГО ГИГАНТА.
335 597992
>>7991
Что сказать-то хотел?
336 598013
>>7986
Если населить его войнами, то почему нет?
337 598014
>>8013
Если эти войны будут людьми, то они будут легко умирать, надо им поддержку, СЖО. Лучше если они были бы андройдами.
image.png137 Кб, 218x218
338 598016
Стикер63 Кб, 512x451
339 598017
>>8016
Кажется, этот ГАЗОВЫЙ ГИГАНТ ВОСПЛАМЕНИЛСЯ.
340 598025
Суп хвощ. А если построить или выкопать на луне охуенных размеров параболическое сопло, а потом подрывать в нем термоядерные боньбы, это ведь будет создавать тягу? Можно таким образом разгонять Луну?
341 598026
>>8025
Да.
Это будет создавать тягу.
Можно таким образом разгонять луну.
Более того, выкапывать ничего и не надо, первые взрывы сами сделают это параболическое зеркало.
342 598028
>>8025
Тягу можно создавать даже высирая говно и кидаясь им в небо на скорости больше второй космической. Вопрос в эффективности.
343 598031
>>7921
Предположим, смогут удержать.
344 598037
>>8031
Так а вопрос смысла не имеет. Максимальная плотность атмосферы зависит от гравитации данного тела, поэтому условия из задачи невыполнимы в принципе. Кроме того, от гравитации зависит и то, сколько кг атмосферы нужно надуть, чтобы достичь данной плотности (в тех случаях, когда это вообще возможно), поэтому так вот просто "меняя" гравитацию, ты меняешь ответ.
345 598047
А что вызывает ядерный распад? Что является именно причиной того что данный изотоп в данный момент распался?
image.png595 Кб, 2624x1243
346 598054
Поясните про квантовую запутанность.
Почему типа измерение одной штуки как-то влияет на другую?
Они же одинаковые вышли изначально, они и будут одинаковые.
И то что мы не знали их состояния лишь говорит про то что мы их не знали, а не про то что они квантовые-хуянтовые.
Вообще вся эта квантовая хуйня кажется полным бредом и наебаловом.
Вот ты посмотрел и типа изменил что-то. А не посмотрев ты результат не узнаешь. А узнав результат ты на оригинал не влияешь.
Хуета какая-то раздутая на ровном месте, так мир не работает.
347 598060
>>8047
В ядрах атомов действует сильное ядерное взаимодействие, которое притягивает частицы. Но против него действует электромагнитное отталкивание протонов, которые одинаково заряжены. Электромагнитное взаимодействие слабее, но не так быстро ослабевает с расстоянием.

Некоторые изотопы находятся "на грани": в общем-то ядро собрано в кучку, но случайные колебания частиц могут нарушить баланс, когда частицы разлетаются чуть дальше чем надо - и пиздец, электромагнитное отталкивание пересиливает, все разваливается.

Ну это так, упрощенно, конечно. На самом деле там еще и слабое ядерное замешано, объясняющее бета-распад, например. Да еще и протоны-нейтроны не элементарные частицы, а состоят из кварков, да еще и квантовые эффекты. Но в целом как-то так.
348 598065
>>8054
Грубо говоря это связано с волновыми свойствами частиц. Когда мы измеряем волну мы определяем где находится корпускула. Когда взаимодействуют 2 волны и мы не можем сказать, где находится корпускула в облаке вероятности у нас получается 2 связанные частицы. И когда мы определяем где находится одна из провзаимодействующих частиц, мы можем отмотать время назад и определить где находится вторая частица через первую.
349 598072
>>8037
Тут ещё смотря из чего атмосфера. Одно дело водород или гелий, и совсем другое какой-нибудь ксеон или радон.
350 598075
>>8072
Да. Там много факторов. Температура вот еще, горячий газ менее плотен. Но так или иначе вопрос бессмысленен, потому что желаемые условия недостижимы.
351 598076
>>8054

>так мир не работает


А как он работает? Желательно с пруфами.
1280px-Bell.svg.png85 Кб, 1280x1097
352 598101
>>8054
О, еще один познакомился с удивительным миром квантовой физики.

>И то что мы не знали их состояния лишь говорит про то что мы их не знали, а не про то что они квантовые-хуянтовые.


А теперь почитай про ЭПР-парадокс, про Джона Стюарта Белла и про его неравенства.

Оказалось, что можно поставить такой эксперимент, который дал бы разный результат в случаях:
1) Если бы у запутанных частиц было конкретное изначальное состояние, просто неизвестное нам — так называемые "скрытые переменные".
2) Если бы никакого начального состояния не было, частицы действительно находились бы в смешанном квантовом состоянии, а конкретные величины появлялись только при измерении, причем одномоментно у обоих запутанных частиц сразу.

Поставили эксперимент, оказалось, что верен вариант 2, состояние появляется только в момент разрушения запутанности. Скрытых переменных нет, локального реализма нет, квантовая физика это и есть реальность, а не просто удобный способ описания нашего незнания.

То, что выросшим на классической механике людям и ОТО, и квантовая физика кажутся каким-то бредом, никого не волнует, реальность под их хотелки подстраиваться не собирается.
353 598126
>>8054

>Вообще вся эта квантовая хуйня кажется полным бредом и наебаловом.


>Вот ты посмотрел и типа изменил что-то. А не посмотрев ты результат не узнаешь. А узнав результат ты на оригинал не влияешь.


Наблюдение невозможно без взаимодействия с наблюдателем, при помощи какой-либо из фундаментальных сил. Собственно, под наблюдением и понимается взаимодействие. Поэтому наблюдение влияет на квантовую систему.
354 598158
>>8101
Как оказалось? Бред полный, они просто не знали результат, никак измерение на другие вещи не может влиять, хватит дичь пороть, это не имеет никакого смысла.
355 598168
>>8158
Аноним, это тред тупых вопросов, а не тупых утверждений. Если ты и так всё сам знаешь, зачем тебе анонимные учёные мирового уровня?
356 598169
>>8168
Чтоб пояснили какого хуя придумали такую ерунду вместо того чтобы честно признать "мы не знаем".
357 598170
>>8169
Тебе уже пояснили, почему наблюдение влияет на квантовую систему, максимально простым способом. Или ты видишь это, или продолжаешь срать своим говном.
358 598171
>>8170
Не пояснили.
"Поставили эксперимент и оказалось" это пояснение уровня яскозал. Нихуя не показано что эта бридятина имеет смысл.
359 598172
>>8171
Вот, читай. >>8126
360 598174
>>8172
Да это понятно, непонятно с хуя ли решили что другая квантово запутанная частица вдруг меняется, а не сразу изначально такими параметрами обладала.
361 598246
>>8174
Потому что никаких параметров не существовало до измерения.
362 598248
>>8246
Тыскозал?
У всех параметры существуют, а у этих с какого-то беспруфного вскукарека оказывается не существует, ага щас.
363 598300
>>7282
Юпидер + Сатурн справа налево
364 598309
Какой максимальный размер и массу для наглядности – в землях может иметь спутник газового гиганта?
365 598318
>>8309
Не больше самого гиганта. Не ближе предела роша.
366 598319
>>8318
И еще внутри сферы Хилла. Чем ближе гигант к своей звезде, тем она меньше.
367 598376
>>8318

>Не больше самого гиганта. Не ближе предела роша.


То есть у того же юпитера может быть спутником юпитер поменьше, диаметром и массой 0,9 от юпитера?
368 598377
>>8376
Это будет двойная планета, а не спутник.
369 598382
>>8377
Будто есть большая разница. Вон тот же Плутон-Харон - вполне себе двойная система, при их разнице в 8 раз по массе барицентр находится над поверхностью Плутона. Тем не менее Харон считается спутником Плутона. Кто меньше - тот того и спутник.
370 598385
>>8382

>Будто есть большая разница.


Номенклатурно есть. Это уже не луна.

>Вон тот же Плутон-Харон - вполне себе двойная система


Да. И никто из них не является планетой.

>Тем не менее Харон считается спутником Плутона.


А Плутон не является планетой.
371 598390
>>8385
Да какая разница-то? Ну будет у тебя Нептун вокруг Юпитера вращаться например, получится то же самое - одновременно и Нептун-спутник-Юпа, и двойная планета Юпитер-Нептун. Если конечно IAU не придумает каких-нибудь охуенных исключений снова. Всё это вопрос номенклатуры и не стоит выеденного яйца, природа не знает этих категорий, у неё это просто сорта крутящихся по разным траекториям говен.
372 598391
>>8390
Хотя ладно, Нептун слишком хилый для Юпа, пусть вон Сатурн обращается, у них меньше разница.
373 598393
>>8390
>>8391
Все уже придумано. Сатурн вокруг юпитера будет двойной планетой, а не луной.
Почитай определения, все расписано.
374 598406
>>8377

>Это будет двойная планета, а не спутник.


Про максимальные размеры и массу спутника мне так и не написали, только то, что спутник должен быть чуть меньше юпитера.
375 598414
>>8385
Всё же я настаиваю, что плутон всё же не планета, а карликовая планета, но планета же пусть и карликовая.
Io.png544 Кб, 813x787
376 598423
Насколько большой может быть каменная планета с твердой поверхностью? Насколько там будет велика гравитация? Может ли такая планета иметь Землю в качестве спутника, а не двойной планеты?
377 598438
>>8423

>в качестве спутника, а не двойной планеты


Нет там четких критериев. А если и назначат, то все это так или иначе довольно произвольно. Барицентр? Ну так Луна, например, удаляется, барицентр, соответсвенно, тоже.

Вот етот >>8390 правильно сказал.
378 598448
>>8423
Газовые гиганты бывают не крупнее Юпитера, а каменистые могут быть чуть больше земли, из-за сжатия от гравитации.
379 598473
>>8423
При приближении примерно к 10 массам Земли каменная планета при формировании натягивает на себя кучу газа и становится ледяным/газовым гигантом наподобие Урана/Нептуна. Если плотность как у Земли, то и гравитация соответствующая будет.

>>8448

>Газовые гиганты бывают не крупнее Юпитера


На самом деле примерно 13 Юпитеров - верхний порог, после которого зажигается реакция и оно становится коричневым карликом.
380 598478
>>8473
Возможен ли ледяной гигант с твердой поверхностью? По которой можно ходить?
Интересна планета, которая могла бы быть вдвое больше Земли в диаметре, но с низкой плотностью и примерно равной гравитацией.
381 598482
>>8478
Нет, слишком дохуя газа, поэтому четкого фазового перехода нема. Хотя может при определенных условиях и возможны какие-то граничные фазы, при которых относительно резкий градиент киселя.
382 598524
Если бы земля была спутником или парной планетой, к какой-либо другой планете, влияло бы хоть как-то на землю, то из чего она состоит при прочих равных?
15217261527960.jpg44 Кб, 720x720
383 598543
>>8524
Это GPT-3? Я не могу распарсить этот коммент своей нейросетью.
384 598576
>>8543
Что такое распарсить?
385 598578
>>8576
Так шизик свои таблетки называет, которые он забыл принять
386 598585
>>8473

>>Газовые гиганты бывают не крупнее Юпитера


>На самом деле примерно 13 Юпитеров - верхний порог


Он, наверное, имел в виду радиус. Там даже если масса растет, радиус остается примерно как у Юпитера (ну чуток больше только) за счет сжатия.
изображение.png623 Кб, 584x1095
387 598586
изображение.png340 Кб, 1280x720
388 598587
>>8585
И вот еще иллюстрация
389 598588
>>8586
Не вижу ничего про размер планеты, только про массу.
390 598592
Сколько пиропатронов в союзе? Какие есть альтернативные способы помимо пиропатрона?
391 598594
>>8592
Кроме пироболта защёлку или замок можно разомкнуть магнитом или пережигаемой ниткой (последний способ используют в основном для фиксации сложенных солнечных батарей). В принципе все эти способы используют в тех или иных случаях для размыкания силовых конструкций.
392 598599
>>8406
Тебе надо прочитать определение планеты.

>>8414
Она не является тем, что называется "планета".
393 598600
>>8438
Вам номенклатуру соблюдать или шашечки?
394 598601
>>8543
Не переживай, я получил достаточно увечий и черепно-мозговых травм чтобы распарсить.

>>8524
Скорее нет, чем да. Все, из чего состоят планеты создано из примерно одного и того же.
С Меркурия сдуло всю пародию на атмосферу, а жиробусы врое Юпитера будучи поотдаль удержали газики. А в целом у всех дофига железа, кремния и кислорода среди прочих. Именно планетообразование, коллизии и дальнейшая жизнь влияет на то что они из себя станут представлять.
395 598603
Можно ли использовать МКС как межпланетную станцию?

Можно ли переоборудовать Шаттл как корабль для инопланетных колонистов?
396 598604
>>8599
"карликовая акула" не является "акулой".
397 598605
>>8604
Ну, правильно. А карлан - не человек.
398 598609
>>8603

> Можно ли использовать МКС как межпланетную станцию?


Нет, она в принципе не предназначена для использования вне низкой орбиты Земли. Рассчитана на регулярное снабжение, связь ближняя, нет должной радзащиты ни у электроники, ни у жилых объёмов, нечем пулять такую массу на межпланетную траекторию и тормозить, не рассчитана на сколько-нибудь заметную тягу (банально батареи с радиаторами сложатся нахрен, да и сама станция), площадь батарей рассчитана на солнечную постоянную у Земли, и т.д. и т.п., тысячи причин.

>Можно ли переоборудовать Шаттл как корабль для инопланетных колонистов?


Шаттл это тоже корабль исключительно для низких орбит. У него было ещё больше ограничений, потому что его пилили для всего и сразу, и всё он делал плохо. Автономности у него нет. У него и термодинамические ограничения были, он в определенных случаях нуждался в земной тени, чтобы охладиться. Войти в атмосферу Земли с межпланетной траектории он тоже не может.
399 598614
Если бы частная компания воссоздала шаттл, как у Наса почти в точности с такими же ттх, они смогли бы получать прибыль и иметь спрос и нишу на рынке сегодня?
400 598624
>>8614
Нет конечно. Даже отбросив в сторону оверинженеренность самого шаттла - у пилотируемой ебалайки в коммерции попросту нет особых задач. Таскать лысых обезьян на каждый пук вообще рискованно и дорого, а уж выход в открытый космос это вообще туши свет, проще робота послать.
401 598640
>>8604
Нерелевантно.
Карликовая планета не планета. Факт. Все, съебись теперь, даун тупорылый, говно ебаное.
image.png164 Кб, 623x343
402 598641
>>8624
А Энергию?
X-33larcbig.jpg19 Кб, 600x480
403 598646
Почему бы не сделать первую ступень ракеты в виде самолета с несущим фюзеляжем? Не нужно ебаться с ховерслэмом и запасать для него топливо, не нужно таскать бесполезные выдвижные крылья, просто взлетел, выкинул вторую ступень, сел даунрейндж как самолет, там кропаль заправился и вернулся обратно на космодром. Или можно даже целую цепочку из космодромов сделать, ракета взлетает, первая ступень садится на следующем космодроме, там на нее надевают вторую ступень, запускают, и так до последнего космодрома, где уже производят ТО и отвозят на первый космодром.
404 598647
>>8646

>кропаль


Чего!?
405 598649
>>8647
Ну децл.
406 598650
Не надоело быть такими задротами? Еще и деньги наверное на эту хуиту тратите. Сходили бы с друзьями забухали, попиздились с кем нибудь после клуба, потом к машке давалке. Вот она жизнь, а нет там где то под хуйпитером и урАНУСом
407 598654
>>8649
Не слушаю его. Он до сих пор активен что ли, зачем вспомнил?
408 598710
Скорость вращения Земли на экваторе немного меньше 500 м/с. Допустим, увеличив трение об эфир магически мы останавливаем Землю за неделю, для этого должно хватить ускорения в 0,001 м/с^2, даже меньше. То есть, само по себе это должно быть охуенно мало, никто и не почувствует (понятно, что моченые засекут сразу же, но смысл в том, чтобы не причинить немедленных неудобств обывателям).

Какие будут последствия? Что пришло в голову: полярное сжатие должно "разгладиться". Как это будет происходить? Предполагаю, будут землетрясения, а какого масштаба, уровня "лол, чот земля дрожит" или "мы все умрем"? Вулканы активизируются? Возникнут ли ураганные ветры, или мы карусель достаточно медленно остановили? Как быстро будет нагреваться дневное полушарие и остывать ночное (день теперь равен году) и до каких температур?
409 598712
>>8710
Полный пиздос биосфере, полный пиздос обоим полушариям, а на терминаторе пиздецовые ураганные ветра.
Это убивает краба.
410 598715
>>8712

>на терминаторе пиздецовые ураганные ветра


Это из-за нагрева? Типа, расширяющийся воздух с горячей стороны несется на место скукожившейся остывшей атмосферы на ночной?
411 598734
>>8641
Ну это уже обычная ракета, у такой задачи как раз есть. Но с коммерческими нагрузками у дорогущего водородного сверхтяжа тоже не особо.
Ты только что Буран. Но если ты имеешь в виду ступень с воздушными движками, а не ракетными - то ты только что воздушный старт, который применяется. Правда профитов у него мало.

ВРД не применяют как полноценный двигатель первой ступени потому, что они эффективны лишь в небольшом параметре высот и скостей, быстро летать не могут, тягу нормальную на протяжении всей траектории хуй дадут.
412 598736
>>8734
почему они кстати пускают эту хуйню с такого редкого гражданского самолета, а не с какого-нибудь б-52?
413 598737
>>8736
А они и запускали с б-52.
Мне кажется, что тристар дешевле и проще в обслуживании, чем военный бомбардировщик.
Ну и плюс триджеты быстро стали ненужным говном после смены разрешений на полеты над атлантикой, когда двуджеты смогли летать.
414 598738
>>8736
Похуй что переделывать. А Б-52 бы не уложился в массогабариты (?), хуй знает на самом деле. Внешняя подвеска все равно нужна была бы. Ну и военный самолёт, а это коммерческая контора, с пегасуса вообще прибыли ждали.
Мясищевцы как-то предлагали пилить хуйню, в которой ракета засовывается прямо в трюм немодифицированного Ан-124 и выкакивается по рельсам в полёте через задний люк, крича "за ДВД!". Интересно было бы посмотреть на такую наркоманию, конечно.
170531-stratolaunch.jpg125 Кб, 1200x675
415 598739
>>8738
Ну и да, вот этот адовый пиздец, который так ничего и не запустил. Интересно, какой ебли стоило не дать двум фюзеляжам разлететься в разные стороны.
Стикер63 Кб, 250x250
416 598742
>>8739
Мне в огурцаче подобную хтоническую конструкцию приходилось обмазывать стяжками что твой Флайер Райтов.
417 598745
>>8738

>А Б-52 бы не уложился в массогабариты (?), хуй знает на самом деле. Внешняя подвеска все равно нужна была бы.


но с него еще
но мог же. с него и сейчас пускают всякие тесты
>>8737
но как может быть проще самолет, который нигде не летает? я понимаю что он им достался скорее всего бесплатно, но обслуга уникального на сегодняшний день самолета явно дороже чем обычного военного
image.png11,9 Мб, 3030x2606
418 598746
>>8738

>А Б-52 бы не уложился в массогабариты (?), хуй знает на самом деле. Внешняя подвеска все равно нужна была бы.


но с него еще
но мог же. с него и сейчас пускают всякие тесты
>>8737
но как может быть проще самолет, который нигде не летает? я понимаю что он им достался скорее всего бесплатно, но обслуга уникального на сегодняшний день самолета явно дороже чем обычного военного
image.png891 Кб, 1200x646
419 598747
>>8745

>но как может быть проще самолет, который нигде не летает? я понимаю что он им достался скорее всего бесплатно, но обслуга уникального на сегодняшний день самолета явно дороже чем обычного военного


Не, тут похую. У Траволты, частного лица, боенх 707, который уже лет 50 не актуален. Части есть или заказываются/изготавливаются, сервисное обслуживание такое же как и у любых рейсовых самолетов по сервисной книжке квалифицированным персоналом.
Военный самолет выйдет дороже в эксплуатации и обслуживании

>>8746
Ты заебал, с первого раза понятно было.
420 598749
>>8745
>>8746
Я думаю что дедуля 52 всё-таки обосрался бы тащить 23 с хером тонны под одним крылом, и надо было бы резать брюхо, как и сделали. Ну и жрёт он небось как не в себя.
421 598892
Достаточно ли в кометах воды и аммиака чтоб накидать на Марс океан и азотную атмосферу?
422 598931
>>8892
Если ты про "терраформирование", то там говорится именно про обстрел планеты подобными телами. Получается, что одной не хватит, а пиздануть здоровой глыбой опасно.
423 598932
>>8931
А хуле случится? Не разъебем же планету
424 598933
>>8932
Ну сначала слишком дохуя просто в космос улетит и еще дохуя на орбите планеты зависнет, уже подлететь сложнее. Потому и по чуть чуть водички подливать самый норм вариант. Ну и кометы как правило объекты не маленькие и на больших скоростях, будет просто 2 планеты, а не одна няшная. Все надо садится и считать и зависит и от планеты выбранной и от того что под рукой для обстрела есть.
425 598934
>>8931
Мне казалось, что просто спамить по северному полушарию этими кометами не крупных размеров, чтоб кора не треснула и кьюриосити с персевиренсом не опрокинуло, так они будут по днищу будущего океана бить и выделившаяся энергия согревать набирающуюся атмосферу и океан.
426 598935
>>8892
Солнечный ветер и радиация снесёт атмосферу за сто лет, так как нет магнитного поля.
427 598936
>>8935
За миллионы.
h0xkkncd4BY.jpg21 Кб, 500x201
428 598938
Вопрос глупый, но занимательный.
Вот услышал такую теорию где то, что мол большой взрыв породил несколько вселенных, не только нашу что видим.
И что есть мол параллельно развивающиеся вселенные и что типа вот эти вот 30% темной материи это оно и есть, но только почему то не видно. Что думаешь анон? Словно в сосиски на тарелке, палок сосисок много и все на одном подносе и МЫ это только одна из сосисок.
429 598939
>>8938
Это не теория.
430 598940
>>8939
Значит чья то догадка. Но все же. Такое возможно? Т.е известное о темной материи позволяет такому быть?
431 598941
Какой мощности должен быть термоядерный хлопок, чтобы сдуло ощутимую часть атмосферы. Скажем, 5%. Или это нереально?
432 598942
>>8940
Ничего не известно.
433 598943
>>8941
Выше тератонны, повредит кору и огромная часть поверхности станет лавой.
434 598944
>>8943
На обратной стороне Земли тоже всем пизда, или как-то можно выжить?
435 598945
>>8942
А ее гравитационное взаимодействие? Но может ли она представлять собой вуаль покрывая то что эту гравитацию и испускает?
436 598946
>>8944
В бункерах.
Поверхность станет практически безжизненной.
437 598947
>>8945
>>8942

>Ничего не известно.


Это не имеет предсказательной силы и не проверяемо, поэтому это ненаучный вопрос.
438 598948
>>8946
А какие-нибудь сейсмические волны бункеры не сжуют? Или все же расстояние слишком большое даже для тератонны?
439 598949
>>8947
Хорошо... А что вообще известно про темную материю кроме как то что тона есть и это так назвали?
440 598950
>>8949
То, что она собирается в кучки размером с галактику.
441 598951
>>8949
То, что она гравитирует и объясняет вращение галактик.
442 598952
>>8948
Зависит от позиции и конструкции бункера. Стабилизированный в горном мегалите бункер вроде НОРАДа выживет.
443 598954
>>8950
>>8951
И в тоже время проходит(пересекается) сквозь нашу галактику, землю и нас людей самих. Я прав? Или это про темную энергию?
444 598955
>>8954
Материя. Да, проходит.
Потому, что не взаимодействует Э/М.
445 598956
>>8955

>Э/М


Электропоезд Мытищинский?...
446 598957
>>8955
>>8956
А... "Энергии на материю"
448 598959
Анон, если бы тебе сейчас предложили бы понаблюдать за каким то одним неизвестным сейчас космическим явлением и получить на него ответ, то что бы ты выбрал?
Например: что за горизонтом событий у черных дыр
449 598960
>>8959
Я бы посмотрел, что происходит за горизонтом событий пизды твоей мамаши.
450 598961
>>8960
Там все тоже самое, что и у твоей.
451 598962
>>8961
Но она же мертва.
452 598963
>>8962
Ну ты же откуда то вылез
453 598968
>>8959
Если это явление неизвестно, то откуда мне знать о нем?
Ты имел в виду "неисследованном"?
Опять же, каким образом я смогу это наблюдать?
Я бы в первую очередь хотел бы видео записать, например, или снять показания спектрографов и прочих приборов, вот это наблюдения.
Воочию всё, что я могу увидеть из астрономических явлений даже будучи бессмертным летакой - это яркий свет.
Я кроме фотонов ничего не вижу, я крайне паршивый способ наблюдения.
454 598981
Итак, аноны. У меня есть одна шизофреническая идея новая конструкция прыжкового ранца, ололо, и у меня такой вопрос: чем окислять керосин/метан? Желательно не криогеника, желательно малотоксичное то бишь, не гиперголик, ибо это само по себе значит высокую реактивность.
455 598982
>>8981
Перекись ебашь, как дедушка Кармак завещал.
456 598996
>>6078 (OP)
Но видос же хуета. Это круглое тело, значит достаточно массивное, больше 1000км размером. А значит вести себя в таких условиях будет как жидкость а не как твердое тело. Тоесть сначала будет происходить деформация - растягивание сферы в каплю. Изза такой деформации будет происходить нехуевый нагрев до тысяч градусов с полным расплавлением тела, и только потом распидорашивание на мелкие куски.
457 599002
>>8981
прыжковый ранец на ракетных двигателях уже говно затея
458 599003
Чем ограничен радиус детектирования слияний ЧД в гравитационном детекторе? Можно ли задетектить слияние на существующих детекторах произошедшее прям у кромки реликтового излучения? Частота ведь та же или нет?
459 599005
>>9003
Дело не в частоте.
460 599006
Допустим, космонавт обосрется в космос. Возможно ли такое, что покак разнесет по разным телам и из покака разовьется жизнь в пригодных условиях?
461 599007
>>9006
О, давно не было. Где пропадал?
462 599008
>>7051
Слышал где-то что консистенция майонеза такая какая есть из-за квантовых эффектов, но не знаю насколько это правда.
463 599009
>>7051
Цвета определяются квантовыми эффектами вроде. Все атомы излучают на определенных длинах и именно потому, что излучают дискретные кванты определенной энергии, а не любую хуйню.
464 599017
>>9003
Нет, не такая же. Любая волна, даже гравитационная, пока ползёт по расширяющейся вселенной, тоже расширяется вместе с ней и теряет частоту и энергию.
465 599020
Есть какая-то инфа по тахионам? Есть ли какие-то явления объясняющиеся их существованием, или они чисто конструкт выведенный из математики?
466 599097
Есть какое нибудь приложение на андроид чтоб за звездным небом наблюдать как нибудь? Созвездие там найти нужное или узнать что за спутник?
467 599098
>>9097
Да, конечно есть.
468 599099
>>9098
Отлично. а какие?
469 599105
>>9099
heavens above
470 599134
>>9105
Я благодаря ему разглядел как-то случайно паровозик старлинков. Но звезды и созвездия там так, для ориентировки - оно заточено именно под спутники больше.
471 599136
>>8981

>Желательно не криогеника,


Селитру расплавь, лол.
472 599140
Имеет ли смысл подогревать топливо перед отправкой в камеру сгорания?
473 599145
>>9140
Оно и так подогревается. Идёт вокруг сопла. Регенеративное охлаждение ептить
474 599152
>>8981

>желательно малотоксичное то бишь, не гиперголик, ибо это само по себе значит высокую реактивность.


Малотоксичные гиперголики существуют. Вот этот >>9136 недалек от них. >>593614 →
475 599154
>>9020

>Есть ли какие-то явления объясняющиеся их существованием


Нет. Тахионы были придуманы физиками-теоретиками, которые баловались на тему "как могла бы выглядеть частица, двигающаяся быстрее света", невзирая на сломанную причинность и прочее подобное, выходящее за рамки мысленного эксперимента. Никакого соответствия реальному миру нет (хотя схожий матаппарат используется в теории поля)

>чисто конструкт выведенный из математики


Это.
image.png560 Кб, 453x604
476 599198
>>9154

>в теории поля


На конечной станции кондуктор осматривает вагоны и в одном видит на лавочке заснувшего студента, а рядом лежит книжка Ландау "Теория поля".
Кондуктор будит студента: "Ну вставай, агроном, приехали!"
477 599219
>>9152
Если гиперголик нетоксичный, проверьте его импульс условия работы.
478 599229
Бля, только сейчас узнал, что газовые гиганты РЕАЛЬНО ГАЗОВЫЕ и у них нету поверхности, мне это аж голову вскружило, до сих пор в башке бардак. Напишите, пожалуйста, годные каналы/сайты про космос на русике.
И пару вопросов!
1) Почему тот же Юпитер держит свою форму и газ с неё нахуй не улетает? Гравитация? А что именно притягивает и держит на одном месте? Как всё это облако вообще по космосу передвигается и не распадается?
2) Почему чорную дыру нельзя увидеть?
Заранее спасибо!
479 599230
>>9229

>Почему тот же Юпитер держит свою форму и газ с неё нахуй не улетает? Гравитация?


А хули с Земли воздух не улетает? Да, гравитация. Гравитирует любое вещество, в том числе и газ. Просто слабее, потому что он не такой плотный, то есть у него меньше масса. Но ведь его дохуя - так что какая разница? Солнце вон вообще из газа (точнее плазмы) состоит и ничего, все здоровы, Солнце не разваливается. Но если тебе будет от этого легче, у Юпитера есть твердое ядро.

2) Почему чорную дыру нельзя увидеть?
Потому что даже свет с нее улететь и вонзиться тебе в глаз не может. Но можно увидеть аккреционный диск вокруг нее, например. Или гравитационное линзирование.
480 599231
>>9229

>газовые гиганты РЕАЛЬНО ГАЗОВЫЕ и у них нету поверхности


У них нету поверхности в том смысле, что там идет плавный переход от газа к жидкости к твердому веществу. Самое обидное, что внутри Юпитера лежит в огромных количествах охуеннейшее ракетное топливо - металлический водород, но его оттуда никак не вытащить.
481 599232
>>9231
Даже если б был способ, дешевле сжать водород заново, чем ковырять тысячи километров Юпитера.
482 599234
>>9230

>А хули с Земли воздух не улетает? Да, гравитация. Гравитирует любое вещество, в том числе и газ. Просто слабее, потому что он не такой плотный, то есть у него меньше масса. Но ведь его дохуя - так что какая разница? Солнце вон вообще из газа (точнее плазмы) состоит и ничего, все здоровы, Солнце не разваливается. Но если тебе будет от этого легче, у Юпитера есть твердое ядро.


Просто удивляет, что по космосу летает охуенно огромный сгусток газа и ничего с ним не происходит. Почему именно такие размеры, почему не больше, почему не меньше? Потому что сила гравитации определённая и не позволяет всему этому улетать или, наоборот, увеличиваться?
А что вообще излучает эту гравитацию?

>Потому что даже свет с нее улететь и вонзиться тебе в глаз не может. Но можно увидеть аккреционный диск вокруг нее, например. Или гравитационное линзирование.


Огромный чёрный сгусток в космосе, который что-то засасывает, не может намекать на дыру? Или вообще не так выглядит? Или такое часто случается, мол, просто пространство без планет?

>>9231

>У них нету поверхности в том смысле, что там идет плавный переход от газа к жидкости к твердому веществу.


К жидкости? Как это вообще выглядит? Если у нас жидкость просто расположена на поверхности, то там просто в воздухе? Просто представить не могу, что на ядре Юпитера расположено дохуища, в несколько раз больше, чем ядро, жидкости. Или не больше?
483 599235
>>9234

>Почему именно такие размеры, почему не больше, почему не меньше?


Больше может быть, причем намного (те же звезды), меньше - вряд ли (если мы не о твердом веществе). И Юпитер без ядра, скорее всего, не смог бы образоваться. То есть, при нынешнем размере твердое ядро ему и не нужно, он бы и так был стабильным. А вот накопить такую массу начиная с газа не получится. Почитай вот это, хотя это больше про звезды, но ближе к тому, что тебя интересует, как я понимаю:
https://ru.wikipedia.org/wiki/Гравитационная_неустойчивость

>Огромный чёрный сгусток в космосе, который что-то засасывает, не может намекать на дыру?


Он нихуя не огромный, ЧД с массой Солнца имела бы радиус порядка 3 км, мы такого размера даже не все астероиды в Солнечной системе знаем (причем мы их засекаем по отраженному свету, а ЧД не отражает). Все известные ЧД от нас дохуя далеко, с такого расстояния они ничем от точки не отличаются. Если б ты был на расстоянии, с которого ЧД выглядит огромной, тебя бы уже убило. Да и вообще, космос довольно черный в среднем.
484 599241
Аноны, а что сейчас с воздушным стартом? С одной стороны размеры спутников падают - часто запускают всякую мелочь. С другой - наши вояки (а еще китайский и США) двигают тазом, всячески изобретая разный гиперзвук и противоспутниковые ёбы, которые как раз по идее и должен запускаться с больших высот.
Нет мыслей объединить или пока все одно дороже обычного пуска выходит?
485 599243
>>9241
а смысл? военным нужна мобильность, в обычных космических запусках хз зачем это нужно
486 599244
Сколько таких же солнечных систем в млечном пути, как наша?
И что в центре галактик? Такое яркое и большое.
487 599245
>>9244

>Сколько таких же солнечных систем в млечном пути, как наша?


Насколько таких же? Вообще звезд несколько сотен миллиардов, но есть же и дохуя непохожих звезд среди них. А у похожих могут быть непохожие системы.

>И что в центре галактик?


Обычно сверхмассивные черные дыры.

>Такое яркое и большое.


Много звезд. Иногда (особенно в относительно молодых галактиках, которые далеко от нас) СМЧД много кушают и светят, что дурные, но обычно это просто много-много звезд, сидящих (относительно) близко друг от друга.
488 599247
>>9245

>Насколько таких же? Вообще звезд несколько сотен миллиардов, но есть же и дохуя непохожих звезд среди них. А у похожих могут быть непохожие системы.


И вокруг этих сотен миллиардов звёзд вращаются сотни миллиадов планет? Или они одиночки?
Как вообще люди узнают, как образуются, из чего состоят и каким законам подчиняются планеты/галактики далеко-далеко-далеко от нашей? Как люди узнают о внешности таких далёких планет, дорисовывают очертания или телескопы настолько мощные?
489 599248
>>9247

>И вокруг этих сотен миллиардов звёзд вращаются сотни миллиадов планет? Или они одиночки?


Ну, в среднем открытых планет на звезду довольно мало. Но вполне вероятно, что это связано в первую очередь с недостатками методов обнаружения. Короче, пока рано говорить, но скорее всего 1-2 планета на звезду уж точно будет (может и значительно больше).

>Как вообще люди узнают, как образуются, из чего состоят и каким законам подчиняются планеты/галактики далеко-далеко-далеко от нашей?


Основной физический принцип: изотропность пространства, то есть что физические законы везде одинаковы. Он принципиально недоказуем, но отлично соответствует всем наблюдаемым явлениям (то есть, опровержений не наблюдается) и общенаучно-философскому принципу бритвы Оккама.

Состав планет определяется спектрометрией, отчасти следует из плотности (если плотность 1 г/см^3, то явно там не железо), хотя для экзопланет это редко можно применить. Из общих теорий гравитации и планетообразования.

>Как люди узнают о внешности таких далёких планет, дорисовывают очертания


Методом тыка, обычно такой хуйней собственно моченые не занимаются. Посмотри на Юпитер, есть основания ожидать, что планеты со сходными параметрами будут выглядеть похоже.

>или телескопы настолько мощные?


Нет. Даже диски удаленных звезд видны в единичных случаях (с крошечным разрешением). О наблюдении диска экзопланет пока и речи нет. Они и открываются в подавляющем большинстве случаев (или во всех?) не прямым наблюдениям, а по "косвенным уликам".
490 599249
>>9248

>Ну, в среднем открытых планет на звезду довольно мало. Но вполне вероятно, что это связано в первую очередь с недостатками методов обнаружения. Короче, пока рано говорить, но скорее всего 1-2 планета на звезду уж точно будет (может и значительно больше).


Планеты тяжело находят из-за их блеклости? Хоспади, если только на 1 галактику несколько сотен миллиадов звёзд, то насколько же огромна вселенная в целом, пиздец просто...

>Нет. Даже диски удаленных звезд видны в единичных случаях (с крошечным разрешением). О наблюдении диска экзопланет пока и речи нет. Они и открываются в подавляющем большинстве случаев (или во всех?) не прямым наблюдениям, а по "косвенным уликам".


То есть человечество знает ничтожно малое количество и состав галактик? Часто видел, как появлялась инфа про чёрные дыры с планетами через овер9000 световых лет, это тоже предположения? Тот снимок чёрной дыры за 50 млн световых лет, это сильно дохуя или дыра сильно аномальна?
Спасибо за ответы!
491 599250
>>9249

>Планеты тяжело находят из-за их блеклости?


Ну, формально да, а также за счет близости к звезде: банально не хватает разрешающей способности различить тусклую точку планеты возле звезды, более яркой на порядки. Но на самом деле прямым наблюдением считай что нихуя не открывают, там все сложнее: https://ru.wikipedia.org/wiki/Экзопланета#Методы_поиска_экзопланет

>То есть человечество знает ничтожно малое количество и состав галактик?


Как раз состав галактик узнать проще: они светятся, так что можно применить спектрометрию.

>Тот снимок чёрной дыры за 50 млн световых лет, это сильно дохуя


Да, там какая-то магия по сведению снимков с телескопов на разных "концах" Земли для получения одного виртуального телескопа в пол-Земли размером. Не спрашивай, я сам тут толком нихуя не понимаю.
492 599251
>>9250
И последнее. Что будет, если Юпитер столкнётся с обычной планетой, вроде Земли, и другим газовым гигантом, вроде Сатурна?
493 599255
>>6078 (OP)
Подскажите документалок с крутыми визуализациями наподобие оп вебмки, но не старые которые еще до 2010 снимали. BBC Планеты все пересмотрел.
Waterphasediagram.gif123 Кб, 1844x1246
494 599259
>>9234

>К жидкости? Как это вообще выглядит? Если у нас жидкость просто расположена на поверхности, то там просто в воздухе?


Фазовую диаграмму представляешь? Просто при определенном давлении и температуре различие между газом и жидкостью пропадает.
https://ru.wikipedia.org/wiki/Сверхкритическая_жидкость
На поверхности Венеры атмосфера находится в том же состоянии, там литералли сверхкритический океан-неокеан из углекислого газа.

>А что вообще излучает эту гравитацию?


Любое тело с массой, даже ты. "Излучает" - неправильное слово, не надо вскрывать эту тему...

>>9231

>Самое обидное, что внутри Юпитера лежит в огромных количествах охуеннейшее ракетное топливо - металлический водород, но его оттуда никак не вытащить.


Блять, у него свойства-то изучены уровня теоретическими писями на воде виляно, для крошечной искусственно созданной в лаборатории песчинки между двумя алмазными наковальнями, о которой до сих пор спорят, нужная ли это фаза. Не надо путать подобные исследования с фактами, и уж тем более не надо делать из этой субстанции ракетное топливо, с таким же успехом можно хотеть звезду отбуксировать в виде источника энергии для движка, а чё - можно же.
495 599260
>>9259

>не надо делать из этой субстанции ракетное топливо


Так он же без юпитерианского давления просто станет газообразным, не? Тоже дело. Другой вопрос как его оттуда высосать и зачем нам СТОЛЬКО водорода именно оттуда, когда это, мягко говоря, далеко не самый редкий элемент в системе.
мимо-анон
496 599261
>>9260
Да просто есть предположение (пока толком ничем не обоснованное, просто теорию крутят), что оно может оказаться метастабильным и не раскукожиться взрывным способом. Недавно статья была.
497 599263
>>9261

>оно может оказаться метастабильным


Но Холмс, если оно метастабильно, то почему же мы его нигде не наблюдаем?
498 599264
>>9251
Они сломаются.
499 599265
>>9251

>И последнее. Что будет, если Юпитер столкнётся с обычной планетой, вроде Земли, и другим газовым гигантом, вроде Сатурна?


1. Земля поглотит Юпитер, сделав его своей атмосферой.
2. Сатурн и Юпитер столкнувшись разлетятся газовыми осколками. На земле возможно будет наблюдать метеоритный дождь из газовых астероидов.
500 599268
>>9230

>Солнце вон вообще из газа (точнее плазмы) состоит и ничего, все здоровы, Солнце не разваливается.


Эт наше. А соберешь звезду массой выше 150 солнечных масс, и каюк.
501 599272
>>9263
Ну, может оно при нагреве раскукоживается, а все природные процессы добычи его из ядра гигантов (типа столкновения с другим гигантом или разрывом его аккреционными силами у ЧД) оным нагревом обязательно сопровождаются.
502 599273
>>9247

>И вокруг этих сотен миллиардов звёзд вращаются сотни миллиадов планет?


Скорей всего.

>Как вообще люди узнают, как образуются, из чего состоят и каким законам подчиняются планеты/галактики далеко-далеко-далеко от нашей?


Science, bitch!

>Как люди узнают о внешности таких далёких планет, дорисовывают очертания или телескопы настолько мощные?


Предположения.
503 599276
>>9272
С учетом температуры и давления в этих самых ядрах гигантов там должна получаться прессованная плазма. Насколько она обладает металлическими свойствами - интересный вопрос для олимпиады по химии, но как британские ученые собираются удерживать её при комнатной температуре - загадка.
504 599296
>>9263
Потому что его никто не достал из Юпитера, очевидно же. А больше нигде в природе таких давлений нет.
505 599373
Если наша галактика столкнётся с другой, то мы ВСЁ?
Миллиарды лет в ращоты брать не будем
image.png3 Мб, 1280x1600
506 599375
>>9373
Не, похую, максимум систему выкинет из галактики, но нам будет ВСЁ из-за Солнца самого.
507 599381
>>9373
Нам будет похуй абсолютно. Вероятность прямого столкновения исчезающе мала, а для того чтобы нарушить планетарную систему, мимозвезде нужно практически внутри пролететь. Что тоже исчезающе маловероятно. Ну и столкновение галактик - процесс пиздец какой медленный.
508 599383
>>9381
>>9375
А если квазар загорится в получившейся йоба-дыре, он не стелиризует галактику?
509 599384
>>9383
Зависит от того как далеко пролететь от него, и какой он светимости. Прям всю галактику - нет.
510 599387
>>9381
Ну, ващет там еще может астероидная бонбардировка начаться из-за относительно близкого прохождения звезды в облаке Оорта, которая нарушит орбиты его тел. Еще, считается что Солнце удачно расположено в области Галактики, где мало молодых взрывающихся звезд, что благоприятно для стабильной жизни, а это может весьма нарушиться при столкновении. Но да, это все из разряда "вероятность рака увеличилась в два раза", а не немедленных катастроф.
511 599388
>>9387

>нарушиться


русский выучи сперва
512 599389
>>9388
может [что сделать?] - нарушиться
Теперь ты.
513 599391
>>9389
нет, меня постоянно хуесосили и я запомнил что мягкий знак может ставится только после шипящих типа ш и щ
514 599392
>>9391
Не трали
515 599394
>>9391
Пиздешь.
516 599396
Можно ли как0то разъебать Юпитер? Условно влетит в него астероид размером с Землю, то он потеряет свою форму? Или поведёт себя как облачко, а затем обратно соберётся?
517 599397
>>9396

>астероид размером с Землю


Эт планета уже.

>то он потеряет свою форму? Или поведёт себя как облачко, а затем обратно соберётся?


Смотря с какой скоростью. На второй влетит - просрется красиво, но целый останется. На 0.99с - на соседних звездных системах новое светило в небе появится.
518 599398
>>9397

>на соседних звездных системах новое светило в небе появится.


Да ну, юпитер не загорится даже если в него жмякнуть землей на световой скорости. Просто ебнет от релятивистских энергий в макромасштабах, без поддержания реакции.
519 599399
>>9398
У атомов земли по 70ГэВ каждого будет ~ 1e-8 джоулей.
На земле 1050 атомов примерно.
Итого 10е42 джоуля испускаемые за секунду.
Это перекрывает 10е26 ватт солнца на 16 порядков.
Более того, это сопоставимо со сверхновой, но не так продолжительно.
Это полностью испарит все планеты солнечной системы сдув большую часть солнца и останется разлетающимся тускнеющим молекулярным облаком плазмы.

Я не говорил, что звезда будет гореть долго.
520 599400
>>9399
Что-то многовато. А кинетическая энергия осколков? Не все же излучится.
521 599401
>>9400
Можешь смело два порядка отбросить, всё равно 1% прореагировавших полностью атомов это 10е40 джоулей.
522 599403
>>9399
Я и говорю - ебнет. А энергии +- такого порядка возникают всегда, когда мы начинаем швыряться объектами макромира на световых скоростях.
523 599404
>>9399
Т.е. если ёбнет по Юпитеру, то сдохнут все?
524 599405
>>9404
Мы - да. Даже если не на той земле, которой ебнули по Юпитеру, а на обычной.
525 599406
>>9404
Тебе же сказали, от скорости зависит. На минимальной будет плюх, клочья газа и новое ядро у юпитера - у него даже радиус особо не вырастет, насчет орбиты не уверен, но скорее всего тоже не особо поменяется. Про 0,99с анон выше все правильно написал.
526 599408
>>9406
Я себя дополню тут - еще и от траектории зависит.
Обычно практически никогда ничего целиком не плюхается отвесно, а значит, что планета попадет в полость Роша и красочно огромными ошметками выпадет в атмосферу Юпитера.
Это тоже посчитать можно, тащемта.
laplace.jpg14 Кб, 280x336
# OP 527 599409
Перекат, что ли:

>>599407 (OP)
>>599407 (OP)
>>599407 (OP)
528 599874
>>9249

>какая огромная вселенная


А какая разница, если мы там все равно никогда не будем. Скорость света и расширение вселенной не даст нам когда-либо покинуть галактику.
Алсо, чтобы охуеть от размеров вселенной и графония рекомендую SpaceEngine, крутая штука.
Тред утонул или удален.
Это копия, сохраненная 27 марта 2021 года.

Скачать тред: только с превью, с превью и прикрепленными файлами.
Второй вариант может долго скачиваться. Файлы будут только в живых или недавно утонувших тредах. Подробнее

Если вам полезен архив М.Двача, пожертвуйте на оплату сервера.
« /spc/В начало тредаВеб-версияНастройки
/a//b//mu//s//vg/Все доски